You are on page 1of 50

 

1. EL TEST DE LA D-XILOSA ALTERADO INDICA:

1. 1. DEFECTO DE LA DIGESTIÓN DE LOS CARBOHIDRATOS.


SÍNDROME DE MALABSORCIÓN POR DEFECTO DE LA PARED DEL
2. 2.
INTESTINO.
3. 3. DÉFICIT CONGÉNITO DE DISACARIDASAS.
4. 4. OBSTRUCCIÓN BILIAR.
Gráfico de respuestas
Comentario

La D-xilosa se absorbe en el yeyuno por difusión pasiva, sin necesidad de que intervengan las
enzimas pancreáticas o las sales biliares. Por ello, está alterada en procesos que afectan a la
pared del tubo digestivo (respuesta 2 correcta), sin verse influida por alteraciones pancreáticas ni
biliares.

El test de la D-xilosa consiste en administrar este azúcar en ayunas, y luego se mide su presencia
en orina, cuando pasan unas horas. Si la encontramos en orina, quiere decir que se ha absorbido a
nivel intestinal.(R/2)
 

2. Un marinero de 37 años acude a consulta por presentar dos úlceras de


aproximadamente l cm de diámetro en el orificio del prepucio. Son dolorosas, de bordes
irregulares y el fondo de las úlceras está cubierto de exudado necrótico. En la exploración
física aparece una adenopatía inguinal izquierda inflamatoria de carácter doloroso y
sensible al contacto. El resto de la exploración es normal. El paciente reconoce haber
mantenido contactos sexuales con prostitutas cada vez que su barco fondeaba en un
puerto, siendo la última vez hace una semana. ¿Cuál sería su diagnóstico?

1. 1. Chancro blando.
2. 2. Linfogranuloma venéreo.
3. 3. Sífilis primaria.
4. 4. Herpes genital.
Gráfico de respuestas
Comentario

Dentro de las ETS, es importante que sepa realizar el diagnóstico diferencial entre ellas. Veamos
opción por opción:

Chancro blando: Produce lesiones múltiples, dolorosas y con adenopatías unilaterales.

Sífilis primaria: El chancro suele ser único, no doloroso, y con adenopatías bilaterales.

Linfogranuloma venéreo: No produce un chancro como tal, sino una lesión papulosa, o incluso
nada en absoluto… Pero después aparecen adenopatías unilaterales de gran tamaño, que acaban
fistulizando.

Herpes genital: Lesiones ulcerosas múltiples, con adenopatías bilaterales y molestias dolorosas.

 
 
 

En este caso, la presencia de una adenopatía unilateral, así como el dolor y el período de
incubación breve (una semana) habla a favor de un chancro blando.(R/1)
 

3. Mujer de 45 años que tras intervención quirúrgica de masa ovárica izquierda presenta
dolor en flanco izquierdo, náuseas, vómitos, ileo paralítico e hidronefrosis izquierda. ¿La
prueba que usted pediría para valorar la lesión que probablemente tenga la paciente,
sería?

1. 1. USG abdominal.
2. 2. USG transvaginal.
3. 3. Arteriografía.
4. 4. Urografía.
Gráfico de respuestas
Comentario

Es una pregunta de dificultad media que debe ser tenida en cuenta. Se trata de un posible ligadura
de uréter que ocasiona hidronefrosis con fiebre, dolor en flanco, náuseas, vómitos e íleo. Se podría
estudiar mediante USG, pero el mejor método diagnóstico en estos casos es la urografía ya sea
intravenosa o retrógrada. En cuanto al tratamiento, la primera mediad debe se la derivación
urinaria.(R/4)

4. Excluyendo la válvula aórtica bicúspide, ¿cuál es la cardiopatía congénita más


frecuente?:

1. 1. Conducto arterioso persistente.


2. 2. Comunicación interventricular.
3. 3. Coartación aórtica.
4. 4. Tetralogía de Fallot.
Gráfico de respuestas
Comentario

Es importante que domine bien su clasificación así como los aspectos clínicos de las principales
cardiopatías congénitas. Recuerde que la cardiopatía congénita más frecuente es la válvula aórtica
bicúspide, seguida de la CIV. Ahora bien, si le preguntan por la cardiopatía congénita cianótica
más frecuente debe contestar la transposición de los grandes vasos, en el RN y la tetralogía de
Fallot a partir del primer año de vida.(R/2)

5. Una de las siguientes NO es causa habitual de hemoptisis:

1. 1. Bronquiectasias.
2. 2. Tuberculosis.
3. 3. Estenosis mitral.
4. 4. Enfisema pulmonar.
Gráfico de respuestas
Comentario
El conocimiento de la semiología en medicina es muy importante porque es necesario dominar
determinados síntomas para así conseguir seleccionar la respuesta correcta en un caso clínico, ya
sea porque lo conocemos o simplemente descartando el resto de las opciones. La hemoptisis se

 
 
 

define como la presencia de sangre en el esputo, normalmente procedente de vías aéreas


subglóticas, y siempre es motivo de preocupación tanto para el enfermo como para el médico. Esta
sangre puede proceder de las vías respiratorias, del parénquima pulmonar o de los vasos
sanguíneos. Las enfermedades del árbol respiratorio pueden ser inflamatorias (bronquitis, que es
la causa más frecuente de hemoptisis, bronquiectasias, fibrosis quística,...) o neoplásicas
(carcinoma broncógeno o carcinoide). Las enfermedades del parénquima pueden ser localizadas
(neumonía, absceso, tuberculosis o Aspergillus) o difusas (sd de Goodspasture y hemosiderosis
pulmonar idiopática). Las enfermedades vasculares que pueden cursar con hemoptisis son el TEP,
las malformaciones y todas aquellas situaciones que aumenten la presión en el árbol pulmonar
(p.e. estenosis mitral). De esta forma vemos que el enfisema no suele producir hemoptisis, por lo
que la seleccionamos como opción correcta.(R/4)

6. De los siguientes, el factor más relacionado con el crecimiento intrauterino retardado


tipo simétrico es:

1. 1. Infección congénita.
2. 2. Hipertensión.
3. 3. Diabetes con vasculopatía.
4. 4. Edad de la mujer por encima de los 35 años.
Gráfico de respuestas
Comentario

Los fetos pequeños para la edad gestacional son aquellos que se encuentran debajo del percentil
10. Se dividen en:

PEQUEÑO PARA EDAD GESTACIONAL CONSTITUCIONAL: no hay alteraciones estructurales,


son normales pero en rango de crecimiento bajo.

PEQUEÑO PARA LA EDAD GESTACIONAL ANOMALO (CIR SIMETRICO): Presentan anomalias


genéticas estructurales o secundarias a una infección TORCH. Son "armónicos". Todos los
parámetros son proporcionados pero retrasados respecto a la edad gestacional.

CIR II O ASIMÉTRICO: Secundarios a una funcion placentaria alterada. Doppler alterado "les falta
comida". Son disarmónicos con BDP y LF acorde con la edad pero DA retardado.(R/1)

7. Lactante de 2 meses y medio, que presenta un cuadro de catarro de vías altas de 2 días
de evolución, acude hoy al servicio de urgencias por rechazo de las tomas y dificultad
respiratoria. En la inspección tiene tiraje subcostal e intercostal moderado, con ruidos
inspiratorios de vías altas; se auscultan roncus y sibilancias diseminadas en ambos
hemitórax; el resto de la exploración es normal. Tiene una saturación de O2 del 90%.
¿Qué patología sospecha?

1. 1. Bronquiolitis aguda.
2. 2. Bronquiolitis obliterante.
3. 3. Cardiopatía congénita descompensada.
4. 4. Asma bronquial.

 
 
 

Gráfico de respuestas
Comentario

Pregunta muy fácil y que no debe fallar, por lo típico y lo frecuente del cuadro clínico. Aunque la
definición de bronquiolitis habla de lactantes menores de dos años con su primer episodio de
dificultad respiratoria, se da más en los menores de 6 meses.

Es importante tener en cuenta los síntomas previos para su diagnóstico, porque en los lactantes
más mayores, sobre todo con antecedentes familiares u otros factores de riesgo para
hiperreactividad bronquial, como la dermatitis atópica, las alergias alimentarias, o el tabaquismo
pasivo, la bronquiolitis puede confundirse con el asma. Las crisis de broncoespasmo, sin embargo,
suelen tener una aparición brusca; en la auscultación, se escuchan fundamentalmente sibilancias
(sin ruidos de secreciones), y lo más importante, las crisis se repiten con frecuencia.(R/1)
 

8. En una postmenopáusica sana, el tratamiento sustitutivo adecuado es:

Antidopaminérgicos tipo veralipride para el síndrome climatérico y asociar


1. 1.
alendronato.
2. 2. Fitoestrógenos.
3. 3. Estrógenos y progestágenos.
4. 4. Asociación de fitoestrógenos con bifosfonatos y calcio con vitamina D3.
Gráfico de respuestas
Comentario

Pregunta sobre el tratamiento de la menopausia que puede resultar algo complicada si no tenemos
los conceptos claros. No hay que confundirse por la variedad de respuestas, ya que en el
enunciado tenemos la clave: postmenopáusica sana. En las opciones 1 y 4 se incluyen
tratamientos para la osteoporosis (raloxifeno y bifosfonatos como el alendronato), con lo que ya no
se trata de mujeres sanas. Los fitoestrógenos son sustancias de origen vegetal con acción
estrogénica débil, con lo que será efectivas en un grupo reducido de pacientes que presenten
síndrome climatérico débil. Es obvio pensar que el tratamiento sustitutivo adecuado es la
combinación de estrógenos y progestágeno, ya que mejora los síntomas del síndrome climatérico y
es útil en la prevención de la pérdida de masa ósea; así, la opción correcta es la nº4.(R/3)
 

9. Señale cuál de los siguientes síntomas NO aparecería en una fractura-luxación del D12
con sección medular, de muy poco tiempo de evolución:

1. 1. Hiperreflexia.
2. 2. Atonía muscular.
3. 3. Pérdida completa de la sensibilidad por debajo de nivel lesionado.
4. 4. Abolición de reflejos superficiales.
Gráfico de respuestas
Comentario

Pregunta difícil y de un tema poco preguntado como es el manejo de los traumatismos medulares.
La sección medular de poco tiempo de evolución se corresponde con el denominado choque
medular, que consiste en la pérdida transitoria de todos los reflejos espinales con parálisis fláccida
por debajo del nivel de lesión (opción 1 incorrecta). Recuerde que el choque medular precede en

 
 
 

un tiempo variable a la lesión medular establecida, caracterizada por signos de 1ª motoneurona por
debajo de la lesión (espasticidad e hiperreflexia).(R/1)

10. Hombre de 62 años que consulta por molestias faríngeas de 3 meses de evolución. A
la exploración se aprecia tumoración ulcero-vegetante de cara laríngea de epiglotis que
infiltra bandas ventriculares. Cuerdas vocales libres con movilidad conservada.
Adenopatías bilaterales múltiples, pequeñas, móviles y no dolorosas. El diagnóstico
anatomopatológico es carcinoma epidermoide moderadamente diferenciado. ¿Cuál de
las siguientes opciones terapéuticas considera más CORRECTA?:

1. 1. Laringuectomía horizontal supraglótica y vaciamiento radical bilateral.


2. 2. Hemilaringuectomía vertical y vaciamiento funcional bilateral.
3. 3. Laringuectomía horizontal supraglótica y vaciamiento funcional bilateral.
4. 4. Hemilaringuectomía vertical y vaciamiento radical bilateral.
Gráfico de respuestas
Comentario
Respecto al tratamiento de los tumores supraglóticos, la cirugía es uno de los tratamientos más
utilizados. El tumor descrito es un tumor supraglótico bajo (supraglotico infrahioideo) que respeta el
plano glotico a nivel de comisura anterior, cuerda vocal y aritenoides y además no paraliza la
cuerda vocal, con lo cual la cirugía a realizar será una laringuectomia horizontal supraglotica. El
estado del cuello es el factor que empeora más el pronostico en estos tumores, ya que las
metástasis regionales cervicales son frecuentes y hace necesario un vaciamiento cervical bilateral
en este paciente que es N2, pero también estaría indicado aunque fuese N0. El vaciamiento si sólo
es un N2 y no afecta a paquete vascular cervical suele ser suficiente con que sea funcional.(R/3)

11. El CO2 es el gas ideal para la realización de un neumoperitoneo en una laparoscopia,


ya que para que un gas sea utilizado en laparoscopia debe ser:

1. 1. Medianamente soluble, irritante e incomburente.


2. 2. Insoluble, comburente e incoloro.
3. 3. Soluble, incoloro e incomburente.
4. 4. Incoloro, insoluble y volátil.
Gráfico de respuestas
Comentario

Se han empleado para el neumoperitoneo diferentes tipos de gases, desde aire, nitrógeno, argón,
helio, CO2 y óxido nitroso. Cada uno de ellos reviste características distintas, pero en términos
generales sólo los gases solubles (CO2 y N2O) tienen aplicaciones en la práctica de la
laparoscopia. A diferencia del N2O, el CO2 es incombustible, evitando el riesgo de chispazos al
usar el electrobisturí.(R/3)

12. Una mujer de 85 años consulta por cansancio y debilidad especialmente por las
mañanas. A veces se encuentra inestable al caminar y tiene que sentarse para recuperar
el equilibrio. En dos ocasiones ha tenido que sentarse para no caer pero niega síntomas
de mareo. Tiene hipertensión arterial, incontinencia urinaria y artrosis. Su tratamiento es
hidroclorotiazida (25 mg/d), oxibutinina (10 mg/d), lisinopril (10 mg/d), calcio (1500 mg/d)
y paracetamol (3 g/d). A la exploración destaca una tensión arterial de 115/70 mmHg,
pulso 80 lpm. Sus movimientos son lentos. Tiene un temblor moderado en las manos.
Puede levantarse de la silla lentamente pero sin necesidad de apoyarse en los brazos.
Camina levemente inclinada hacia delante con poco balanceo de los brazos. Gira

 
 
 

lentamente pero sin perder el equilibrio. No es capaz de mantenerse sobre un solo pie.
¿Cuál de las siguientes posibles actuaciones realizaría en primer lugar?

1. 1. Valorar la agudeza visual.


2. 2. Estudio con mesa basculante.
3. 3. Medir la tensión arterial tumbada y levantada.
4. 4. Intento terapéutico con L-dopa.
Gráfico de respuestas
Comentario

Pregunta difícil.

El caso clínico nos describe a una mujer de 85 años con clínica probablemente debida a una
enfermedad de Parkinson (cansancio, debilidad, inestabilidad con la marcha, bradicinesia, temblor
distal, escaso balanceo con la marcha…). Sin embargo, la paciente consulta por cansancio y
debilidad que aparecen “especialmente por las mañanas”, y además “ha tenido que sentarse para
recuperar el equilibrio y para no caer” negando síntomas de mareo.

Estos datos nos hacen sospechar un cuadro de hipotensión postural en el contexto de una posible
enfermedad de Parkinson. Además, la presencia de tratamiento antihipertensivo, especialmente los
diuréticos, hablan a favor de esta sospecha. También toma un anticolinérgico, que puede estar
contribuyendo (oxibutinina). La clave de esta pregunta, viene dada por el enunciado final: nos pide
la actuación en primer lugar.

Las respuestas 1 y 2 son claramente falsas. La disminución de la agudeza visual no produce este
cuadro, no existen datos de focalidad neurológica que nos hagan sospechar que una RM aportará
información al cuadro, y la mesa basculante es un test realizado para el diagnóstico diferencial del
síncope vasovagal con riesgo de hipotensión grave y asistolia que no debe realizarse en ancianos.

La respuesta 4 es también falsa, aunque lleva a confusión. Es cierto que al tratarse de una anciana
con clínica parkinsoniana el intento con L-dopa es una buena actitud a llevar a cabo. Sin embargo,
en la pregunta se nos pide la primera actuación, y en el contexto de una posible hipotensión
postural, es prioritario descartar esta y tratarla, disminuyendo por ejemplo la dosis de diurético.
Además, el tratamiento antiparkinsoniano con L-dopa puede empeorar o causar hipotensión
ortostática, haciéndose menos recomendable en este cuadro.

Por tanto, la respuesta correcta es la 3, medir la tensión arterial tumbada y levantada, siendo este
el test más fácil y rápido para confirmar el ortostatismo.(R/3)

13. A 4-year-old boy with a history of sickle cell anemia and splenic infarcts becomes to
the emergency department presenting with fever and malaise. Laboratory results shows
anemia, reticulocytosis and leukocytosis. What would be expected in a peripheral blood
smear?

1. 1. Howell Jolly Bodies.


2. 2. Heinz bodies.
3. 3. Dacryocites.
4. 4. Hipersegmented PMN.
Gráfico de respuestas
Comentario
 
 
 

Howell-Jolly bodies. Howell-Jolly bodies are histopathological findings of basophilic nuclear


remnants (clusters of DNA) in circulating red blood cells.They usually indicate spleen damage. In
patients affected by sickle cell anemia hyperviscosity causes spleen infarcts so its function gets
markedly reduced.(R/1)

14. ¿Cuál de los siguientes NO es un factor asociado a candidiasis?

1. 1. Hemorragia intermenstrual.
2. 2. Diabetes mellitus.
3. 3. Corticoterapia.
4. 4. Antibióticos.
Gráfico de respuestas
Comentario

Si se fija la respuesta 2 o 3 puden causar inmunosupresión celular, por lo que en estos pacientes
se incrementa el riesgo. El uso de antibióticos aumenta las probabilidades de candidiasis por
destruir la flora bacteriana habitual. La respuesta que debe elegir es la 1.(R/1)

15. Con la clasificación de choque hipovolémico, por ATLS, señale verdadero o falso: ( 1
) Pérdidas de sangre Clase I, son equivalentes a la donación sanguínea. ( 2 ) En Clase II,
hay marcado compromiso como taquicardia, taquipnea e hipotensión arterial. ( 3)
Pérdidas Clase III, pueden ser manejadas solo con solución salina. ( 4 ) Pérdidas Clase
IV, se pierde aproximadamente 40% de la volemia y ponen en riesgo la vida del paciente.

1. 1. FFVV.
2. 2. VFFV.
3. 3. FVFV.
4. 4. VFVF.
Gráfico de respuestas
Comentario

La opción correcta es la 2 ya que:

-Clase I: V. Las pérdidas son equivalentes a la donación al ser menores de 750ml.

-Clase II: F. No hay marcado compromiso cardiovascular ya que la TA es normal y la taquicardia no es muy marcada.

-Clase III: F. Se deben administrar cristaloides y concentrados de hamatiesen este nivel.

-Clase IV: V. Las perdidas son del 40% del volumen o mayores con riesgo vital(R/2)

 
 
 

16. Médico traumatólogo de 40 años que en una serología rutinaria se le detecta niveles
de antiHBs de 30 UI/I. Refiere vacunación completa según pauta estándar frente hepatitis
B hace 4 años. Sería recomendable:

1. 1. Reiniciar pauta vacunal (0-1-6 meses).


2. 2. Reiniciar pauta vacunal (0-1-2-12 meses).
3. 3. No revacunar.
Darle inmunoglobulina anti-hepatitis B y reiniciar pauta vacunal (0-1-2-12
4. 4.
meses).
Gráfico de respuestas
Comentario

El paciente presenta niveles protectores de AcHBs (>10UI/l) por lo que no requiere más
inmunización frente a la hepatitis B.(R/3)

17. A 43-year-old woman with a


history of multinodular goiter comes to the clinic for a follow-up examination after a
gammagraphy has been performed, which is shown in the image. Laboratory studies
show: TG and TPO antibodies slightly elevated, TSH level 0.001 microUi/ml (normal range:
0.35-4.95), free T4 3.569 ng/dl (normal 0.700-1.480) and free T3 > 30 pg/ml (normal 1.71 -
4.53). Which of the following is the most likely diagnosis?

1. 1. Graves disease.
Functioning metastatic follicular carcinoma of the thyroid gland. Thyroglobulin
2. 2.
levels should be measured.
3. 3. Toxic multinodular goiter.
4. 4. Toxic thyroid adenoma.
Gráfico de respuestas
Comentario

Para acertar esta pregunta, dependemos bastante de la imagen gammagráfica adjunta. En


cualquier caso, no resulta difícil darse cuenta de que se trata de un adenoma tóxico (tiroides
prácticamente abolido, con una enorme hipercaptación en una zona redondeada). Este diagnóstico
encaja perfectamente con el perfil hormonal que nos describen (elevación de las hormonas
tiroideas con TSH suprimida, es decir, el de un hipertiroidismo). Si se tratara de un bocio
multinodular tóxico, la captación se vería en forma de múltiples nódulos, como su nombre indica.

 
 
 

Tampoco podría tratarse de un estruma ovari, donde tendríamos una gammagrafía hipocaptante
(es un tumor ovárico que secreta hormonas tiroideas y por ello la TSH estaría suprimida). Es difícil
pensar en una metástasis de un cáncer folicular, ya que estamos a nivel del tiroides, donde en todo
caso sería un tumor primario… Y, aun así, sería muy difícil pensar en un cáncer, por tratarse de un
nódulo “caliente”. Por último, la enfermedad de Graves-Basedow produciría una hipercaptación
tiroidea, pero difusa.(R/4)

18. En un paciente que presenta una eritrodermia, los siguientes signos clínicos harían
sospechar una etiología psoriásica EXCEPTO uno, señálelo:

1. 1. Límite neto de las lesiones.


2. 2. Exfoliación seca de todo el tegumento, incluido el cuero cabelludo.
3. 3. Estado general conservado.
4. 4. Prurito marcado en las localizaciones que la psoriasis afecta típicamente.
Gráfico de respuestas
Comentario

Se trata de un pregunta complicada, que exige un conocimiento bastante concreto de las


características de un tipo de psoriasis no muy preguntada en el ENARM.

La eritrodermia psoriásica no es uno de los apartados más preguntados.

Debe recordar al menos: - definición de eritrodermia: enrojecimiento generalizado, con


descamación que no representa una entidad en sí, si no que refleja una presentación clínica de
diferentes entidades. - En este caso nos preguntan por la causa psoríasica de la misma. La
psoriasis representa en 20% del total de las causas de eritrodermia. Suele precederse de lesiones
típicas de psoriasis en placas y suele ser también el resultado del cese de tratamientos como el
metotrexate o corticoides sistémicos. - Clínicamente puede orientar a su diagnóstico: o cambios
unguelaes o descamación plateada en placas delimitadas en cara, rodillas y codos no de forma
generalizada.(R/4)

19. ¿Cuál de las siguientes situaciones NO predispone al Kernicterus en el recién


nacido?:

1. 1. Ingesta materna de sulfamidas.


2. 2. Sepsis neonatal.
3. 3. Hipoalbuminemia neonatal.
4. 4. Alcalosis metabólica.
Gráfico de respuestas
Comentario
El kernicterus es la consecuencia del depósito de bilirrubina indirecta sobre los ganglios basales
del SNC. Entre sus causas se necesita una situación de hiperbilirrubinemia muy importante (debida
a hemólisis, sepsis, hemorragia interna, TORCH) y factores adicionales (como prematuridad,
acidosis metabólica, hipoxia, enfermedad de base, ingesta materna de sulfamidas,
hipoalbuminemia,...)(R/4)

20. De las siguientes características, una de ellas NO es característica de la intolerancia


a la lactosa:

 
 
 

1. 1. Lactato fecal aumentado.


2. 2. Heces acuosas y con olor ácido.
3. 3. Deposiciones explosivas.
4. 4. Cuerpos reductores (Clinitest) negativos en heces.
Gráfico de respuestas
Comentario

La intolerancia transitoria a la lactosa es un cuadro autolimitado en el tiempo que aparece tras una
gastroenteritis invasiva , debido a la destrucción de los enterocitos.

La incapacidad de digerir la lactosa hace que este azúcar permanezca en heces. Al ser la lactosa
un azúcar reductor, se acidifican las heces, apareciendo diarrea explosiva y eritema anal por
irritación, aumentando el lactato fecal y apareciendo en el Clinitest presencia mayor del 0.5% de
cuerpos reductores, es decir, test POSITIVO, por eso la respuesta 4 es la única falsa.(R/4)

21. Señale en qué supuesto hay que estudiar siempre la causa de la ictericia:

1. 1. La ictericia que aparece a las 48 horas de vida.


2. 2. La bilirrubina alcanza una cifra superior a 10 mg/dL.
3. 3. La bilirrubina directa supera el 20% de la total en cualquier momento.
4. 4. En el prematuro alcanza un valor superior a 12 mg/dL al 7º día de vida.
Gráfico de respuestas
Comentario

Esta pregunta puede parecer tediosa y compleja, pero no lo es tanto si tiene las nociones claras.
Ha de saber que las hiperbilirrubinemias pueden ser de dos tipos en función de la cifra de
bilirrubina directa: colestáticas (si BID excede el 20%) y no colestáticas (si BID menor del 20%). La
opción 3 designa una ictericia colestática, tributaria por tanto de estudio. El resto de las opciones
forma parte del espectro de la ictericia fisiológica.(R/3)

22. La edad más frecuente de convulsión febril es:

1. 1. 1 mes - 4 años.
2. 2. 6 años - 10 años.
3. 3. 3 meses - 5 años.
4. 4. 7 dias - 5 años.
Gráfico de respuestas
Comentario

Es simplemente un dato memorístico, o la sabe o no. La respuesta correcta es la número 3: 3


meses a 5 años.(R/3)

23. Un RN perderá calor por radiación si:

1. 1. Esta expuesto al aire.


2. 2. No se le seca correctamente.
3. 3. Sus ropas están muy frías.
4. 4. Se le desnuda en un ambiente frio.
Comentario
La reanimación neonatal es un tema que debe dominar.

 
 
 

Pregunta sencilla. La respuesta correcta es la 4. Repase la siguiente lista.

Evaporación: a través de piel y pulmones.

Conducción: hacia sólidos que están en contacto.

Radiación: hacia sólidos que no hacen contacto.

Convección: diferencia de temperatura con el aire circundante.(R/4)

24. Marque V ó F, respecto a la meningoencefalitis bacteriana: ( ) La glucosa aumenta en


el LCR, ( ) Las proteínas aumentan en el LCR, ( ) Los gérmenes ingresan al encéfalo por
los plexos coroideos, ( ) Para disminuir el edema cerebral usamos los corticoides, ( ) La
ceguera como complicación es muy frecuente.

1. 1. F V V F F.
2. 2. F V F V V.
3. 3. V V V V F.
4. 4. F F F F F.
Gráfico de respuestas
Comentario

La meningitis bacteriana se caracteriza por:

- Disminución de la glucosa en el LCR.

- Proteinas aumentadas en el LCR.

- Los gérmenes entran por los plexos coroideos.

- No se usan corticoides para reducir el edema cerebral

- No es frecuente la ceguera como complicación sino la sordera.(R/1)

25. Un paciente alcohólico, vagabundo, acude a Urgencias por síndrome febril de larga
evolución y deterioro del estado general. A la exploración el paciente está sucio y
descuidado y presenta un soplo sistólico en foco aórtico. Se realiza un ecocardiograma
que demuestra la presencia de una vegetación en la válvula aórtica, a las dos semanas
los hemocultivos continúan negativos. Este paciente presenta una endocarditis por:

1. 1. Bartonella quintana.
2. 2. Streptococcus bovis.
3. 3. C. albicans.
4. 4. S. aureus.
Gráfico de respuestas
Comentario

 
 
 

La endocarditis infecciosa (EI) es un tema muy importante en el examen. Lo más preguntado es la


etiología, clínica, tratamiento y profilaxis.

Nos describen una EI en la que los hemocultivos son negativos, y de las 4 respuestas la más
probable es la Bartonella quintana (RC- 1), que se ha descrito con mayor frecuencia en alcohólicos
indigentes, cono el paciente de este caso. El cuadro se conoce como “fiebre de las trincheras”.

Otros microorganismos que producen endocarditis infecciosa con hemocultivos negativos son los
bacilos del grupo HACEK, que precisan medios de cultivo enriquecidos e incubación durante un
tiempo prolongado.

Coxiella y Brucella son también gérmenes productores de EI con hemocultivos negativos, suelen
afectar a la válvula aórtica y, con frecuencia, precisan cirugía para su tratamiento.(R/1)

26. Niño de 3 años, previamente sano, en el cual al examen físico presenta fiebre, cefalea,
signos meningeos y fotofobia, cual sería la etiología más probable:

1. 1. Glaucoma agudo.
2. 2. Neuritis óptica.
3. 3. Meningitis.
4. 4. Rabia.
Gráfico de respuestas
Comentario

No puede equivocarse en esta pregunta. La respuesta correcta es la 3 meningitis por presentar


fiebre, cefalea, fotofobia y signos meningeos.(R/3)

27. ¿Cuál de las siguientes opciones caracteriza la fase aguda de las dermatitis
eccematosas?:

1. 1. Eritema, edema y vesículas.


2. 2. Formación de costras y descamación.
3. 3. Aparición de estrías de Wickham.
Pápulas y placas eritematosas bien delimitadas y cubiertas por varias capas de
4. 4.
escamas finas y nacaradas.
Gráfico de respuestas
Comentario

El eccema es una reacción cutánea inflamatoria que se caracteriza histológicamente por


espongiosis, acantosis más o menos marcada y un infiltrado linfohistiocitario perivascular
superficial. La espongiosis es el edema intercelular de la epidermis que conduce a la ruptura de las
uniones intercelulares con la formación de vesículas. Los eccemas pueden dividirse, desde el
punto de vista evolutivo, en tres fases. En la fase aguda predomina la espongiosis y la formación
de vesículas. Clínicamente esto se traduce en la presencia de eritema, edema, vesículas, ampollas
y exudación. En la fase subaguda la espongiosis y las vesículas disminuyen mientras aumenta la
acantosis. Además se forma un estrato córneo paraqueratósico. En la clínica se observan lesiones
costrosas y descamación. Finalmente en la fase crónica se encuentra hiperqueratosis con áreas de

 
 
 

paraqueratosis y acantosis que clínicamente se manifiesta como grietas, fisuras y liquenificación.


En el liquen plano se forma un reticulado blanquecino en la superficie de las pápulas que se
denomina "estrías de Wickham". La opción 4 corresponde a las lesiones de la psoriasis.(R/1)

28. Una mujer de 55 años acude a consulta por presentar rectorragia con heces de
aspecto mucoide. En el examen endoscópico se encuentra una tumoración plana de
superficie vellosa, de 4 x 3 cm, que se reseca endoscópicamente aparentemente de forma
completa. ¿Cuál es la actuación médica prioritaria en la paciente?

Hacer un estadiaje tumoral inmediato, sobre todo con el fin de descartar


1. 1.
metástasis hepáticas precoces.
Realizar como tratamiento inmediato resección intestinal de la zona que incluya
2. 2. el lecho tumoral, precedida de quimioterapia neoadyuvante y radioterapia
preoperatoria.
Realizar un estudio histológico exhaustivo de la pieza con el fin de descartar áreas
3. 3.
de carcinoma intramucoso o infiltrante.
Realizar un estudio clínico, radiológico y genético urgente, con el fin de descartar
4. 4.
poliposis múltiple familiar y cáncer colorrectal hereditario.
Gráfico de respuestas
Comentario

Estamos ante una mujer de edad media con rectorragia y el hallazgo de una tumoración que se
reseca. Ante todo, la actitud prioritaria será filiar el tipo de lesión con el que nos encontramos, ya
que de ello dependerá nuestra actitud posterior (opción 3 correcta). Lo primordial a descartar es la
existencia de focos de malignidad, que no puede presuponer de antemano (opciones 1 y 2
incorrectas). Por último, en general las poliposis familiares no debutan en este tipo de edades y
hay otros datos clínicos añadidos para ponernos tras la pista (opción 4 incorrecta).(R/3)
 

29. En una mujer embarazada de 18 semanas, sin antecedentes médicos de interés, con
correcto seguimiento de la gestación y sin patología de interés durante las primeras 18
semanas, con niveles de beta-HCG dentro de la normalidad, PAPP-A dentro de la
normalidad, glucoproteína b-1 específica de la gestación en valores de normalidad,
detectamos una alfafetoproteína sérica elevada. ¿Cuál es la malformación congénita que
más probablemente detectemos en el feto?

1. 1. Atresia esofágica.
2. 2. Onfalocele.
3. 3. Mielomeningocele.
4. 4. Gastrosquisis.
Gráfico de respuestas
Comentario

La elevación de la alfafetoproteína se relaciona estrechamente con las alteraciones del tubo neural,
como el mielomeningocele (respuesta 3 correcta). El resto de las opciones también producirían
elevación de esta proteína, pero si nos preguntan la malformación congénita más probable,
debemos marcar las de carácter neurológico.(R/3)

 
 
 

30. Sobre la atención inmediata del RN, marque la CORRECTA:

1. 1. El score APGAR determina la conducta a seguir en el RN.


2. 2. La aspiración traqueal siempre es mandataria y obligada.
3. 3. La dosis de vitamina K es: 0.1 mg intramuscular.
4. 4. El APGAR expresa la buena adaptación vital del RN a la vida extrauterina.
Gráfico de respuestas
Comentario

Respuesta 1 falsa. El Apgar no determina la conducta a seguir en el RN.

Respuesta 2 falsa. No siempre habrá que realizar aspiración traqueal. Tiene indicaciones precisas.

Respuesta 3 falsa. La dosis de vitamina K es de 0.5 a 1 mg IM.(R/4)

31. ¿Cuál de los siguientes NO es factor de riesgo de osteoporosis?:

1. 1. Obesidad.
2. 2. Multiparidad.
3. 3. Tabaco.
4. 4. Raza blanca.
Gráfico de respuestas
Comentario

Pregunta muy fácil sobre la osteoporosis en el climaterio y sus factores de riesgo. Se puede
responder de forma directa, señalando como opción correcta la 1, ya que las mujeres con
obesidad, al tener mayor panículo adiposo, poseen mayor cantidad de aromatasa, que es la
enzima encargada de la conversión periférica de los andrógenos en estrona (estrógeno principal en
el climaterio). Por ello, la obesidad puede considerarse incluso como un factor de protección.

La raza blanca está más predispuesta a padecer osteoporosis (opción 4), al igual que las
fumadoras (el tabaco produce hipoestronismo, opción 3), y las multíparas (opción 2) relacionado
con el estado estrogénico.(R/1)

32. ¿Cuál es la vía de diseminación más frecuente del cáncer de ovario?:

1. 1. Implantación directa en cavidad abdominal.


2. 2. Vía linfática.
3. 3. Vía hematógena.
4. 4. Transporte a través del tracto genital.
Gráfico de respuestas
Comentario

La vía de diseminación más frecuente del cáncer de ovario es por contigüidad; en este caso,
implantación directa en la cavidad abdominal.(R/1)

33. Un recién nacido a término nacido por cesárea presenta dificultad respiratoria,
taquipnea e imagen de "redistribución" de los vasos pulmonares, diafragmas aplanados
y líquido en las cisuras. No hay hipoxemia, hipercapnia, ni acidosis; ¿cuál es el proceso
más probable?

 
 
 

1. 1. Enfermedad de la membrana hialina.


2. 2. Neumomediastino.
3. 3. Taquipnea transitoria del recién nacido.
4. 4. Síndrome de Wilson-Mikity.
Gráfico de respuestas
Comentario

La taquipnea transitoria es la modalidad de dificultad respiratoria típica del RN término o casi


término. Para su génesis, se requiere además un desencadenante obstétrico (parto por cesárea o
vaginal rápido), que hace que el tórax del RN no sufra compresión en el canal del parto, con lo que
el líquido pulmonar no sería expulsado. Precisamente, ese líquido pulmonar en la radiología y la
evolución favorable dan la clave para llegar al diagnóstico. Respuesta 3 correcta.(R/3)

34. A 5-year-old badly behaved child is brought to the pediatrician's office by his mother,
because he has very poor school performance. She has noticed that often the boy stares
blankly at the wall for a few minutes. He has a normal physical and neurological
development. What test should be performed next?

1. 1. EEG study.
2. 2. Holter ECG study.
3. 3. Serological blood tests.
4. 4. Lumbar puncture.
Gráfico de respuestas
Comentario
EEG study. The reported symptoms are compatible with absence seizures. Absence seizures are
defined by a brief loss and return of consciousness, generally not followed by a period of lethargy.
In this case, consciousness gaps are responsible for poor school performance. Valproic acid is an
effective treatment. A complete medical history and physical examination is the most important
diagnostic tool, but EEG is also useful.(R/1)

35. Ante una imagen mamográfica de microcalcificaciones agrupadas y moldeadas, sin


tumoración mamaria palpable, ¿cuál es el diagnóstico más probable?:

1. 1. Tumor filodes.
2. 2. Fibroadenoma.
3. 3. Carcinoma intraductal.
4. 4. Carcinoma intralobulillar infiltrante.
Gráfico de respuestas
Comentario

Pregunta sumamente importante para el ENARM. Lo más importante de esta pregunta es que los
signos mamográficos descritos le hagan sospechar un cáncer de mama. La presencia
demicrocalcificaciones agrupadas nos obliga a descartar este diagnóstico, por lo que habrá que
obtener una muestra histológica.

Observe que en el enunciado dicen: “sin tumoración mamaria palpable”. Si hubiese un nódulo
claramente definido, la forma de obtener la muestra podría ser mediante una PAAF… Pero si no se
palpa nódulo alguno, realizar una PAAF sería como puncionar a ciegas. Por ello, en un caso así,
habría que emplear una biopsia guiada con arpón.(R/3)

 
 
 

36. Mujer de 32 años de edad con AP de trastorno bipolar en tratamiento crónico con litio.
Ingresa en psiquiatría con síntomas de fase maniaca, aunque refiere buen cumplimiento
terapéutico. Durante el ingreso se cursa interconsulta a nefrología por poliuria de 7-8
litros al día con polidipsia y nicturia. Exámenes de laboratorio sin alteraciones.
Osmolaridad en orina 140 mosm/l. La sospecha diagnóstica y la actitud más adecuadas
serán:

Diabetes insípida por litio. No precisa pruebas diagnósticas de confirmación; se


1. 1.
debe suspender cuanto antes tratamiento con litio.
2. 2. Potomanía. No precisa otras pruebas diagnósticas ni actitud terapeútica alguna.
Probable tubulopatía por ingesta crónica de litio. Está indicado realizar un test de
3. 3.
deshidratación.
Tubulopatía proximal por litio. Suspender tratamiento con litio tras descartar
4. 4.
potomanía con un test de deshidratación.
Gráfico de respuestas
Comentario
Es importante recordar que la ingesta crónica de litio se encuentra entre las causas más frecuentes
de diabetes insípida nefrogénica adquirida. En este caso concreto y dado que la paciente presenta
descompensación de su trastorno psiquiátrico de base, estaría indicado hacer el diagnóstico
diferencial con la potomanía, haciendo un test de deshidratación. (R/3)

37. A 2-day-old infant has a markedly increased cranial perimeter (30 cm). He weighs 2400
gr and thorough physical examination reveals: multiple bilateral cervical and inguinal
lymphadenopathies, a bulging fontanel, hepatosplenomegaly and opacities in both eyes.
Cardiac auscultation shows a systolic murmur in the second left intercostal space. His
mother had a febrile episode during her first trimester of pregnancy but she did not seek
medical advice. What is the most likely diagnosis?

1. 1. Perinatal CMV infection.


2. 2. Congenital rubella.
3. 3. Birth-acquired herpes.
4. 4. Congenital Toxoplasmosis.
Gráfico de respuestas
Comentario

Ante un neonato que presenta antecedente de retraso del crecimiento intrauterino, hiperplasia
retículo-endotelial (hepatoesplenomegalia, adenopatías), anemia y trombopenia, debemos
considerar una posible infección connatal. Este niño presenta patología ocular (leucocoria,
probablemente por cataratas), cardiopatía (soplo) y microcefalia. De los tres datos que integran la
tríada de Gregg (cardiopatía, cataratas y sordera neurosensorial), nos han mencionado dos, y el
resto del cuadro es compatible, por lo que el diagnóstico más probable corresponde a una rubéola
congénita.(R/2)

38. En el desarrollo de venas varicosas en la extremidad inferior el factor MENOS


importante es:

1. 1. Antecedentes familiares positivos.


2. 2. Arteriosclerosis obliterante.
3. 3. Edad.
4. 4. Sexo.
Gráfico de respuestas
 
 
 

Comentario

Pregunta de dificultad moderada.

Estas venas pueden ser primarios o secundarias.

Las primarias resultan de un desarrollo defectuoso de las paredes venosas con debilidad congénita
de las mismas.

Las varices secundarias se producen por insuficiencia venosa profunda y por incompetencia de las
venas perforantes.

La etiología del proceso varicoso no está clara, aunque se conocen una serie de factores que
predisponen y favorecen la aparición de las mismas. El factor más importante es la postura.
Aproximadamente la mitad de los pacientes con varices tienen antecedentes familiares. Otros
factores son aquellos que dificultan el flujo del retorno venoso como el embarazo, la obesidad o las
masas tumorales. Las varices aumentan con la edad y son mucho más frecuente en mujeres.(R/2)

39. Un niño acude a consulta de control al mes de edad y tiene las siguientes medidas:
talla 52 cm, peso 4 Kg y perímetro cefálico 37 cm, correspondiendo dichas medidas al
percentil 40. Si el niño recibe lactancia materna exclusiva y es un niño sano, usted
esperaría en el siguiente control mensual:

1. 1. Que solo la talla persista en el percentil 40.


2. 2. Que el peso suba al percentil 90.
3. 3. Que todas sus medidas continúen en el percentil 40.
4. 4. Ninguna de las anteriores.
Gráfico de respuestas
Comentario

Es un paciente que se encuentra dentro de la normalidad, por lo que en el control mensual


siguiente continuará en dicho percentil.(R/3)

40. Una paciente de 30 años, con un cuadro clínico de sangrado menstrual irregular,
dismenorrea, dispareunia y una esterilidad de 3 años de evolución, presenta exámenes
de laboratorio hormonales normales y un ultrasonido transvaginal que informa de un
útero normal y múltiples formaciones quísticas ováricas bilaterales de 4 cm con signos
de sospecha. Teniendo en cuenta el diagnóstico más probable, ¿cuál de las siguientes
es de elección para obtener el diagnóstico de sospecha?:

1. 1. Ultrasonido abdominal.
2. 2. TC abdominopélvica.
3. 3. Laparoscopia.
4. 4. Culdocentesis.
Gráfico de respuestas
Comentario

Ante un posible caso de endometriosis, dada la edad de la paciente y la sintomatología, en cuyo


estudio, no tenemos las cosas del todo claras, con unos datos hormonales que no están alterados,

 
 
 

lo mejor que podemos plantear es la laparoscopía exploradora, para realizar un diagnóstico de


certeza y podremos tomar biopsias para un posterior estudio anatomopatológico. (R/3)

41. Paciente con antecedente de alcoholismo que ingresa en urgencias por melenas de 5
días de evolución. En la endoscopía se observan várices esofágicas y gástricas grado III
con signos de hemostasia reciente. En un ultrasonido abdominal se aprecia una masa en
cabeza de páncreas compatible con pancreatitis crónica que se confirma en el TAC. Un
estudio de las presiones portales demuestra que existe hipertensión portal no
dependiente del hígado. Para evitar nuevos episodios de sangrado digestivo,
recomendaría:

1. 1. Propranolol.
2. 2. TIPS.
3. 3. Resección pancreática.
4. 4. Derivación portosistémica quirúrgica.
Gráfico de respuestas
Comentario

Nos encontramos ante el caso clínico de un paciente con antecedentes de alcoholismo, sin
diagnostico previo de hepatopatía o pancreatitis crónica, que debuta con un episodio de
hemorragia digestiva alta y en cuyo estudio descubrimos la existencia de várices esofágicas y
gástricas. Lo habitual sería atribuir el origen de éstas a una hepatopatía evolucionada con
hipertensión portal. Sin embargo, el estudio posterior demuestra que ésta no depende del hígado y,
por otra parte, existe una masa en cabeza de páncreas que provoca hipertensión portal
(probablemente por compresión a nivel de la porta). Es decir, tenemos una causa orgánica que
justifica el origen de la hipertensión portal, y que por otro lado no podemos descartar que sea una
neoplasia. Así pues, nos decidiremos por una técnica quirúrgica resectiva.(R/3)

42. Una de las siguientes entidades se asocia a fibrosis retroperitoneal, mediastínica y


colangitis esclerosante:

1. 1. Tiroiditis de Riedel.
2. 2. Tiroiditis de Hashimoto.
3. 3. Carcinoma papilar de tiroides.
4. 4. Carcinoma medular de tiroides.
Gráfico de respuestas
Comentario
Las tiroiditis son procesos en los que existe una inflamación de la glándula. En muchas de ellas el
patrón funcional es de una primera fase de hipertiroidismo por liberación de hormonas tiroideas
preformadas al torrente sanguíneo y una segunda fase de hipotiroidismo que puede ser
autolimitada. La tiroiditis fibrosante o de Riedel consiste en una fibrosis del cuello que puede
afectar al mediastino y al retroperitoneo de forma concomitante, en la que la función tiroidea suele
ser normal, existiendo hipotiroidismo hasta en el 25% de los casos, pero el hipertiroidismo no está
descrito en esta entidad.(R/1)

43. Una niña de 1 año de edad presenta fiebre alta, vómitos y llanto al miccionar, por lo
que se sospecha infección de vías urinarias. En este caso la decisión inicial más
adecuada es:

1. 1. Solicitar examen completo de orina tomado con bolsa colectora.

 
 
 

2. 2. Pedir urocultivo tomado mediante cateterismo vesical o punción suprapúbica.


3. 3. Indicar ultrasonido de vías urinarias.
4. 4. Iniciar tratamiento antibiótico.
Gráfico de respuestas
Comentario

Ante la sospecha alta de infección de orina, como en este caso, se puede obviar el sedimento de
orina con muestra procedente de bolsa y hacer directamente el sondaje, ya que si el sedimento por
bolsa sale alterado, lo que es muy probable, la tendremos que sondar igualmente, y así ganamos
tiempo ante una niña con fiebre alta y vómitos, por lo tanto con bastante malestar.(R/2)

44. Entre las contraindicaciones ABSOLUTAS del tratamiento fibrinolítico en el contexto


del infarto agudo de miocardio, NO estaría incluido:

1. 1. Antecedentes de hemorragia intracraneal.


2. 2. Hipertensión arterial.
3. 3. Ictus isquémico durante los tres últimos meses.
4. 4. Infarto agudo de miocardio debido a disección aórtica.
Gráfico de respuestas
Comentario
Contraindicaciones absolutas de tratamiento fibrinolítico: 1. Hemorragia activa (excluida la
menstruación). 2. Cualquier hemorragia intracraneal previa. 3. Ictus isquémico en los últimos 3
meses. 4. Lesión cerebrovascular estructural conocida o neoplasia intracraneal. 5. Sospecha de
disección aórtica. La hipertensión arterial es una contraindicación relativa, porque podemos
intentar controlarla previamente al tratamiento de reperfusión.(R/2)

45. ¿Cuál es el agente causal más frecuente de bronquiolitis en los lactantes?:

1. 1. Virus sincitial respiratorio.


2. 2. Virus de la gripe.
3. 3. Streptococcus penumoniae.
4. 4. Adenovirus.
Gráfico de respuestas
Comentario

Esta pregunta sobre la etiología de la bronquiolitis es muy sencilla e importante.

Recuerde que el virus respiratorio sincitial es el agente causal principal , aunque otros virus como
el virus de la parainfluenziae, adenovirus, influenza, rinovirus y, de forma infrecuente, M.
pneumoniae, se han asociado a esa enfermedad.(R/1)

46. Embarazada de 37 semanas a la que se le practica monitorización fetal no estresante


con el siguiente resultado: frecuencia cardíaca fetal 135 lpm, ondulatoria normal,
movimientos fetales escasos, sin ascensos en la frecuencia cardíaca y sin
deceleraciones. ¿Qué actitud tomaría?

1. 1. Inducción del parto.


2. 2. Amnioscopía.
3. 3. Amniocentesis tardía.
4. 4. Prueba de Pose.

 
 
 

Gráfico de respuestas
Comentario

Para valorar el bienestar fetal en el tercer trimestre (37 semanas de gestación tenemos varias
opciones, pero en este caso la más acertada será realizar un RCTG estresante o prueba de Pose.

•   No estaría indicado una inducción de parto ni una cesárea urgente, dado que todavía no
queda claro si existe sufrimiento fetal.
•   No estaría indicado realizar una amnioscopía dado que existe riesgo de rotura de
membranas y en principio no tenemos intención de inducir el parto.
•   La amniocentesis tardía se suele realizar en la semana 32 para medir la madurez pulmonar
fetal, medir la bilirrubina en sospecha de isoinmunización Rh, evacuadora en caso de
polihidramnios o amnioinfusión en oligoamnios o tratamiento de la transfusión feto-fetal de
gemelares. Este tipo de amniocentesis no se usa por tanto para el diagnóstico
prenatal.(R/4)

47. Mujer de 70 años que desde hace tres meses presenta vómitos y dolor abdominal que
no cede con la comida. Se acompañaba de pérdida de peso y anorexia. A la EF: palidez
mucocutánea y ligero dolor a la palpación epigástrica. En los exámenes de laboratorio se
detecta Hb 8.9 g/L, VCM 70, amilasa 45 u/L. El test del aliento para H. pylori fue positivo.
En el ultrasonido abdominal se observaron adenopatias en territorio gástrico. Para esta
paciente, ¿cual de las siguientes pautas recomendaría en primer lugar?:

1. 1. Gatroduodenoscopia.
2. 2. Pantoprazol 8 semanas y posterior evaluación.
3. 3. PAAF de adenopatias dirigido por TAC.
4. 4. Anticuerpos anti H. pylori.
Gráfico de respuestas
Comentario

En este caso, con un síndrome general evidente, dolor en epigastrio y anemia microcítica, la
primera sospecha diagnóstica debe ser una tumoración gástrica. Por tanto, es necesaria una
prueba de imagen (gastroduodenoscopia) y, posteriormente, una confirmación histológica, con
biopsia de la lesión. Tanto por la edad de la paciente, como por la positividad del test del aliento
para Helicobacter pylori, lo más probable es que se trata de un adenocarcinoma gástrico de tipo
intestinal.(R/1)

48. Femenino de 60 años de edad, que presenta sangrado genital escaso por orificio
cervical, con ecografía que indica hiperplasia endometrial. ¿Cuál será la conducta más
apropiada?

1. 1. Reevaluación ecográfica.
2. 2. Histeroscopia - biopsia.
3. 3. Laparotomía.
4. 4. Histerectomía abdominal.
Gráfico de respuestas
Comentario

 
 
 

La hiperplasia endometrial es un trastorno que consiste en la proliferación del endometrio por


acción de los estrógenos sin el efecto compensador de la progesterona. Es una enfermedad que
se da fundamentalmente en mujeres con ciclos anovulatorios.

El diagnóstico es anatomopatológico. Ante la sospecha de patología endometria por la presencia


de clínica (metrorragia perimenopáusica o posmenopáusica) o por hallazgos ecográficos
(engrosamiento endometrial mayor de 5 mm en mujeres posmenopáusicas o superior a 12 mm en
mujeres premenopáusicas), es obligatorio obtener matieral para estudio anatomopatológico.

El método diagnóstico de elección es la histeroscopia-biopsia, ya que ofrece la ventaja de


visualizar la cavidad a la hora de realizar la biopsia. Otros métodos también útiles son el legrado
fraccionado o las cánulas flexibles tipo Cornier o Pipelle.

Entre las opciones disponibles, la más adecuada es la opción 2, siendo la 3 y la 4 demasiado


invasivas sin haber obtenido un diagnóstico previo.(R/2)

49. En cuanto al manejo del recién nacido prematuro, señale lo INCORRECTO:

El uso de bicarbonato en prematuros extremos condiciona la presentación de


1. 1.
hemorragia intracerebral.
Los recién nacidos prematuros requieren menores volúmenes de líquidos para
2. 2.
infusión intravenosa que los recién nacidos a termino.
3. 3. Una natremia de 150 mEq/L sugiere disminuir la infusión de líquidos.
4. 4. Todos los recién nacidos de bajo peso son prematuros.
Gráfico de respuestas
Comentario

No todos los RN con bajo peso son prematuros

Se ha asociado a la hemorragia intraventricular con el uso de bicarbonato, por lo que la opción 1


no la debe de seleccionar. Por obvias raones en una hipernatremia el tratamiento es diinuir la
infusión de suero fisiológico e iniciar glucosado o al 0.45%.(R/4)

50. Paciente de 34 años con antecedentes personales de cólicos biliares que se encuentra
en el 7º mes de gestación y que acude a urgencias por presentar dolor abdominal
localizado en cuadrante superior derecho de aproximadamente 24-36 h de evolución, tras
el cual ha presentado dos vómitos, y que actualmente presenta fiebre de 38ºC. Tras ser
valorada por Ginecología sin apreciarse patología ginecológica es remitida a su servicio
de urgencias. ¿Qué actitud diagnóstica seguiría?:

1. 1. Exploración física. Exámenes de sangre y Rx de abdomen.


2. 2. Exploración física. Exámenes de sangre y orina y Rx de abdomen.
3. 3. Exploración física. Exámenes de sangre y orina y ultrasonido abdominal.
4. 4. Exploración física y Exámenes de sangre y orina.
Gráfico de respuestas
Comentario

 
 
 

Pregunta deducible, que debe contestar porque aunque dude con el diagnóstico hay varias
opciones que puede descartar y por tanto: hay que arriesgarse!. Para contestar bien esta pregunta
debe recordar que la urgencia quirúrgica más frecuente en embarazadas es la apendicitis aguda.
No le debe despistar la localización del dolor, ya que a medida que progresa la gestación el
apéndice se localiza en una posición más alta y más lateral, lo cual puede dificultar el diagnóstico.
La apendicitis es un diagnóstico fundamentalmente clínico y se apoya en exámenes de sangre (en
la que lo más frecuente es encontrar una leucocitosis con desviación izquierda). En caso de una
mujer embarazada, por ser el diagnóstico dudoso suele estar indicado la realización de una técnica
de imagen como es un ultrasonido que es inocuo tanto para la madre como para el feto. La Rx de
abdomen y el TAC NO estan indicada en embarazadas.(R/3)

51. Cuál de los siguientes enunciados, NO es criterio para considerar que un neonato
presenta ictericia fisiológica:

1. 1. Incremento diario de bilirrubina > 5 mg/dl.


El nivel máximo de bilirrubina en el RNT se da entre el 2-3 día de vida y en el
2. 2.
RNPT entre el 4-6 día de vida.
3. 3. La bilirrubina indirecta no debe llegar en ningún momento a ser > 15 mg/dl.
4. 4. Una de las causas es el aumento de la circulación enteropática de bilirrubina.
Gráfico de respuestas
Comentario

Las características de la ictericia no fisiológica son las siguientes :

Inicio en las primeras 24 horas de vida.

Duración superior a 10-15 días.

Bilirrubina total mayor de 12 mg/dl en RNT o mayor de 15 mg/dl en RNPT.

Incremento de la bilirrubin a superior a 5 mg/dl/24 horas.

Bilirrubina directa mayor a 1 mg/dl o superior al 20% de la bilirrubina total.(R/1)

52. Referente a la crioglobulinemia mixta esencial. Todo es cierto, EXCEPTO:

1. 1. Se relaciona con crioglobulinas IgG e IgM monoclonal.


2. 2. Puede ocasionar GN rápidamente progresiva.
3. 3. Los niveles de complemento son normales.
4. 4. La erradicación del proceso infeccioso es de utilidad en el tratamiento.
Gráfico de respuestas
Comentario

Esta pregunta es de dificultad media. La opción 1 es correcta y no debe dudarla. La clínica renal
aparece en un 50% de los casos en forma de hematuria y/o proteinuria, aunque también puede
ocasionar una GN rápidamente progresiva (opción 2). La opción 4 es correcta porque se ha
demostrado que el tratamiento de la hepatitis puede ayudar al tratamiento de la vasculitis. Por
último la opción 3 es INCORRECTA porque precisamente los niveles de complemento suelen estar
descendidos por la existencia de inmunocomplejos con IgG e IgM que lo activan.(R/3)

 
 
 

53. A 71-year-old woman presents to her physician with a 4-month history of asthenia and
arthralgia and a 2-month history of nasal congestion and a 2-week history of cough. She
is especially worried now because she has been coughing up blood-tinged sputum for
the last 4 days. These symptoms are not accompanied by fever, nausea or vomiting. She
has hypertension and migraine. Over the past 4 weeks, she has been taking over-the-
counter NSAIDs for her arthralgia. Her pulse is 90/min, breathing frequency are 12/min
and blood pressure is 140/83 mm Hg. On physical examination, the patient appears
fatigued, with diffuse tenderness in her joints which are not warm or erythematous. A
clear nasal discharge is also noted, with no other significant findings. Lab tests show:
Hematocrit 32%, Mean corpuscular volume 82 μm3, Leukocyte count 15,100/mm3,
Segmented neutrophils 71%, Eosinophils 2%, Lymphocytes 15%, Monocytes 12%,
Platelet count 325,000/mm3, Serum Urea nitrogen 32 mg/dL, Creatinine 3.4 mg/dL,
Antinuclear antibodies 1:256, Rheumatoid factor negative, Antineutrophil cytoplasmic
antibodies positive. Urinalysis shows: Blood 3+, Protein 3+, RBC 15–19/hpf, WBC 1–5/hpf,
RBC casts rare. Which of the following is the most likely causative mechanism of this
patient's renal failure?

1. 1. Tuberculosis.
2. 2. Interstitial nephritis.
3. 3. Hypertension.
4. 4. Vasculitis.
Gráfico de respuestas
Comentario
Vasculitis. The combination of pulmonary (blood-tinged sputum), upper airway, arthralgia and
kidney failure in a patient with anti-neutrophil cytoplasm antibodies (ANCAs) is suggestive of
VASCULITIS, particulary GRANULOMATOSIS WITH POLYANGIITIS a.k.a WEGENER'S DISEASE
(if the antibodies react against proteinase= c-ANCA) or CHURG STRAUSS (if the antibodies are p-
ANCA). Remember that the presence of such antibodies is not always detected in patients with
these vasculitides.(R/4)

54. Estamos estudiando a una paciente de 45 años, con antecedente personal de vitíligo,
que en un control de laboratorio de rutina ha presentado cifras de GOT de 75 U/ml y GPT
de 102 U/mL; inmunoglobulina G 1,800mg/dL; serología para hepatitis víricas tipo B y C
negativas. La paciente no refiere hábitos tóxicos. Se realiza estudio de anticuerpos que
nos pone sobre la pista de una hepatitis autoinmune, que se confirma mediante biopsia.
En relación con la hepatitis autoinmune, indique cuál de las siguientes le parece
CORRECTA:

1. 1. Es una enfermedad benigna con escasa morbilidad.


La hepatitis tipo I, además de presentarse con títulos de anticuerpos antinucleares
2. 2.
elevados, presenta serología positiva para el virus C.
3. 3. Los autoanticuerpos anti-LKM-2 se presentan en el tipo II.
Los mecanismos inmunitarios humorales parecen los responsables de las
4. 4.
manifestaciones extrahepáticas de la enfermedad.
Gráfico de respuestas
Comentario

La hepatitis autoinmune tipo 1 cursa con ANA y anti-músculo liso positivos. En la tipo 2, aparecen
anticuerpos anti- LKM 1 y anti-citosol hepático 1. No obstante, aunque no conozca estos detalles,
la respuesta 4 resulta bastante lógica. Si se trata de una enfermedad autoinmune en la que se

 
 
 

implican autoanticuerpos, resulta lógico pensar que las manifestaciones a distancia tendrán alguna
relación con este mecanismo.(R/4)

55. Which of the following statements is CORRECT in relation to iron metabolism?

1. 1. Iron absorption occurs in the ileum.


2. 2. Most of the serum iron is bound to ferritin.
3. 3. Food intake worsens absorption of oral iron.
4. 4. Approximately 20% of the ingested iron is absorbed.
Gráfico de respuestas
Comentario

La absorción del hierro se realiza en el duodeno y los tramos proximales del yeyuno, y se ve
favorecida por el medio ácido. De modo que, con la digestión de los alimentos que aumenta el pH
o con fármacos antiulcerosos, la absorción está disminuida. Además, la proporción de hierro
procedente de la dieta que se absorbe es de un 10%.(R/3)

56. ¿Cuál de los siguientes esteroides tópicos es el de mayor potencia?:

1. 1. 17 alfa hidrocortisona.
2. 2. Propionato de clobetasol.
3. 3. Butirato-propionato de fluometasona.
4. 4. Valerato de Betametasona.
Gráfico de respuestas
Comentario
Pregunta compleja, de estudio en profundidad de Segunda Vuelta. Los Esteroides Tópicos, en
Dermatología, se dividen en tres grupos según su potencia. Muy potentes: tratamientos en palmas
y plantas y dermatosis crónicas, hiperqueratósicas. El más potente de todos es el Propionato de
clobetasol (respuesta 3 correcta) y otros menos potentes, también incluidos en este grupo, como el
Dipropionato de beclometasona. Moderadamente potentes: los más utilizados. Por ejemplo,
Butirato- propionato de fluometasona, Valerato de betametasona, Acetónido de flucinolona. Poco
potentes: utilizados en niños y zonas de piel fina, cara, pliegues. El prototipo es la
Hidrocortisona.(R/2)

57. After slight pain in his right eye for two days, an otherwise asymptomatic 28-year-old
man comes to the emergency department. He bought steroids eye drops at the pharmacy
because his eye itched, using it as required during the previous week. After staining with
fluorescein, examination reveals a branch-like lesion in lower central cornea. Which of
the following is the most appropriate treatment for this patient’s eye lesion?

1. 1. Maintain steroid eye drops treatment until symptoms cease.


2. 2. Add pilocarpine eye drops to the treatment with steroids.
3. 3. Start acyclovir eye drops therapy.
Suspend treatment with steroids. Start with acyclovir and pilocarpine eye drops
4. 4.
therapy.
Gráfico de respuestas
Comentario

 
 
 

La presencia de una imagen fluoropositiva de forma ramificada es muy sugestiva de que el


paciente presente una queratitis herpética. Esta posibilidad se ve muy reforzada por el hecho de
que el paciente se haya aplicado un corticoide tópico durante los días previos. Los corticoides por
su efecto inmunosupresor pueden favorecer la reactivación del virus herpes, y por ello están
contraindicados en esta paciente. También está contraindicada la pilocarpina porque al inducir un
espasmo del músculo esfínter de la pupila, acentúa el dolor. (R/3)

58. En una TAC abdominal realizada a una paciente de 42 años para estudio de un
traumatismo se detecta una imagen de masa renal derecha de 2.5 cm de diámetro máximo
y aspecto graso. Refiere encontrarse asintomática. El resto de la TAC es estrictamente
normal. Señale la FALSA:

Puede aparecer de forma esporádica, lo cual es cuatro veces más probable en las
1. 1.
mujeres.
Surge en la esclerosis tuberosa, en cuyo caso es múltiple, bilateral, de mayor
2. 2.
tamaño y con probabilidad de causar hemorragias espontáneas.
3. 3. La biopsia es fundamental en el diagnóstico.
Las principales complicaciones del AML renal son la hemorragia retroperitoneal
4. 4.
y la hemorragia en el sistema colector urinario.
Gráfico de respuestas
Comentario
El angiomiolipoma (AML) es un tumor mesenquimatoso benigno. Representa en torno al 1 % de los
tumores extirpados quirúrgicamente. La ecografía, TC y RM confirman a menudo el diagnóstico
debido a la presencia de tejido adiposo. La biopsia rara vez resulta útil. El resto de las opciones
otorgan datos fundamentales para el estudio.(R/3)

59. Señale la respuesta FALSA respecto a la fisiología renal:

Aclaramiento de una sustancia es el volumen de plasma que se limpia de una


1. 1.
sustancia determinada al pasar por el riñón en la unidad de tiempo.
2. 2. Es normal una proteinuria de hasta 150 mg/día.
3. 3. La aldosterona actúa fundamentalmente en el túbulo proximal.
4. 4. Una FENa menor de 1 indica fracaso renal funcional.
Gráfico de respuestas
Comentario

Esta pregunta entraña una cierta complejidad, pero debe dominar todos estos conceptos para
defenderse con soltura ante las preguntas de nefrología. Analicemos cada una de las opciones:

1. La definición de aclaramiento de una sustancia corresponde a lo que dice el enunciado.

2. La proteinuria tiene esos límites antes de ser patológica: 150 mg/día.

3. Intenta aprender la localización de los principales transportadores de iones en la nefrona, tanto


para entender el funcionamiento de ciertos diuréticos como la fisiopatología de algunas
enfermedades, las tubulares hereditarias y otras. Si es necesario, haga un dibujo de una nefrona y
píntelos. Es en el túbulo distal donde actúan tanto la ADH como la aldosterona, regulando la
volemia y el agua eliminada. La aldosterona estimula un transportador que reabsorbe Na+ y
elimina K+ e H+, ése es el transportador que está alterado en la enfermedad de Liddle.

 
 
 

4. La excreción fraccional de sodio (aclaramiento de sodio dividido entre el de creatinina ) menor


de uno indica que el fracaso renal es funcional, en esos casos el organismo apenas aclara sodio,
intenta reabsober todo el sodio posible para así aumentar la volemia.(R/3)

60. ¿Qué actitud adoptaría ante un paciente ingresado por presentar un infarto de
miocardio y que presenta una taquicardia ventricular sostenida, aunque sin deterioro
hemodinámico?:

1. 1. Amiodarona.
2. 2. Masaje cardíaco.
3. 3. Bretilio.
4. 4. Verapamil i.v.
Gráfico de respuestas
Comentario

Se trata de una arritmia ventricular primaria. Si produjera deterioro hemodinámico, al tratarse de


una situación de riesgo vital se debería proceder a una desfibrilación eléctrica. Como el estado
hemodinámico es bueno, se prefiere intentar primero el tratamiento farmacológico con amiodarona.
En caso de que no fuese eficaz, se procedería a la cardioversión eléctrica.(R/1)

61. Un paciente portador de una prótesis mitral metálica va a someterse a una resección
prostática. ¿Cuál de las siguientes le parece la profilaxis más adecuada?:*

1. 1. Ampicilina + gentamicina.
2. 2. Ampicilina.
3. 3. Amoxicilina + gentamicina.
4. 4. Ampicilina + metronidazol.
Gráfico de respuestas
Comentario

En esta pregunta es importante saber que, entre los pacientes que precisan profilaxis de la
endocarditis bacteriana ante determinados procesos, se incluyen los portadores de cardiopatías
adquiridas, y dentro de ellos, están los que llevan prótesis intracardiacas como la prótesis mitral de
este caso clínico. En segundo lugar hay que recordar que la profilaxis en procedimientos
genitourinarios es con ampicilina 2g i.v. y gentamicina 1.5mg/kg de peso i.v o i.m. 30 min antes del
procedimiento, seguidos de ampicilina 1g, 6 horas después. Para los procesos odontológicos
recuerda que la profilaxis se realiza con ampicilina.

Aunque recientemente se han publicado las guías americanas, en las que ya no se recomienda
profilaxis antimicrobiana en procedimientos gastrointestinales, ni genitourinarios.(R/1)

62. La enterotoxina colérica es responsable del cuadro diarreico característico de esta


enfermedad. ¿Cuál es su mecanismo de acción?:

1. 1. Efecto citotóxico directo sobre el enterocito.


2. 2. Provocando la activación continua de la adenilato ciclasa.
3. 3. Por activación del GMP cíclico intracelular.
Provocando la liberación de citoquinas por las células del sistema inmune
4. 4.
intestinal.
Gráfico de respuestas
Comentario

 
 
 

Pregunta directa sobre el mecanismo fisiopatológico de producción de diarrea del Vibrio cholerae.
Vibrio cholerae, agente causal del cólera, es una bacteria productora de enterotoxinas, las cuales
actúan directamente sobre la superficie de los enterocitos, sin destrucción de la mucosa, alterando
el intercambio iónico y favoreciendo el paso de agua libre hacia la luz intestinal (a través de la
activación continua de la adenilato ciclasa) por eso genera tanta deshidratación para ocasionar una
diarrea acuosa sin productos patológicos y sin leucocitos al microscopio.(R/2).

63. Ante un paciente con demencia de reciente comienzo y una serología treponémica
positiva en sangre, ¿qué prueba complementaria considera esencial para establecer el
diagnóstico de neurosífilis?

1. 1. EEG.
2. 2. RM craneal.
3. 3. TC craneal con contraste.
4. 4. Análisis del LCR.
Gráfico de respuestas
Comentario

El VDRL, que es una prueba serológica no treponémica, se utiliza para diagnosticar la neurosífilis.
Lógicamente, el estudio se realizará en LCR. Este examen consiste en buscar determinados
anticuerpos, llamados reaginas, en el líquido cefalorraquídeo. Cuando se trata de una sífilis
primaria o secundaria, esta prueba sería negativa. Sin embargo, en una neurosífilis, el resultado
sería positivo.

Recuerde que las pruebas serológicas para la sífilis se dividen en dos grupos:

- Reagínicas: VDRL y RPR, menos sensibles y específicas. Pueden negativizarse con la


curación de la enfermedad, por lo que tienen gran valor para el seguimiento.

- Treponémicas, como el FTA- Abs. Su sensibilidad y especificidad son mayores. Sin


embargo, una vez que son positivas, ya nunca negativizan. Por ello, no tienen valor para el
seguimiento de un paciente, ni para diagnosticar una reinfección.(R/4)

64. La operación de Whipple consiste en la exéresis de:

Parte distal del estómago, duodeno, cabeza de páncreas, vesícula, vía biliar distal
1. 1.
y yeyuno proximal.
2. 2. Vesícula, vía biliar distal y cabeza de páncreas.
3. 3. Vesícula, vía biliar distal y parte distal gástrica.
4. 4. Vesícula, vía biliar proximal y distal y cabeza de páncreas.
Gráfico de respuestas
Comentario
Se trata de una pregunta específica sobre técnica quirúrgica y en concreto sobre la intervención de
Whipple o duodenopancreatectomía cefálica, que es la técnica quirúrgica de elección con intención
curativa indicada en el cáncer de cabeza de páncreas. En esta técnica se resecan el antro gástrico,
todo el marco duodenal y unos cms del yeyuno proximal en bloque junto con la cabeza del
páncreas que se secciona a nivel del llamado cuello pancreático sobre la porta y que incluye la vía
biliar principal distal, es decir el colédoco distal. Como gesto añadido se debe resecar la vesícula
biliar, lo cual es mandatorio siempre que se practica una intervención en la vía biliar principal. Así

 
 
 

pues debemos reconstruir haciendo tres anastomosis a saber: 1) anastomosis entre el conducto de
Wirsung y el tubo digestivo 2) anastomosis entre la vía biliar y el tubo digestivo y 3) reconstrucción
del tránsito digestivo mediante anastomosis entre el estómago y el tubo digestivo. En general todas
estas anastomosis se realizan sobre un mismo asa de yeyuno.(R/1)

65. Una paciente, diagnosticada de infección crónica por VHB en fase replicativa y en
tratamiento con interferón alfa-2b, consulta porque, a las ocho semanas de iniciado el
tratamiento, está más cansada y un estudio de laboratorio demuestra que hay un ligero
aumento de la bilirrubina y de las transaminasas. ¿Qué consideraciones se haría?

1.     1.   Probablemente  tiene  una  hepatitis  de  otra  causa.  


2.     2.   Se  debe  añadir  esteroides  al  tratamiento.  
3.     3.   Probablemente  está  haciendo  la  seroconversión  anti-­‐HBe.  
4.     4.   Probablemente  el  paciente  no  se  está  poniendo  el  interferón.  
Gráfico de respuestas
Comentario

En la hepatitis crónica B en fase replicativa, el objetivo del tratamiento con IFN alfa no es la
negativización del virus (esto sucede en muy pocos casos), sino el paso de fase replicativa a no
replicativa. Cuando esto ocurre, se debe a que el IFN alfa ha conseguido potenciar la respuesta
inmune del paciente frente al virus, siendo ésta suficiente para pasar conseguir el objetivo
propuesto. Esto es lo que se llama seroconversión anti-HBe, y clínicamente suele acompañarse de
un aumento transitorio de la bilirrubina, transaminasas y ligero cansancio. Es como una hepatitis
aguda, pero más leve, traduciendo la potenciación de la respuesta inmunológica frente al VHB, con
la consiguiente inflamación transitoria a nivel hepático.(R/3)

66. Paciente de 34 años con esterilidad


primaria de 3 años de evolución sometida a tratamiento inductor de la ovulación para
ciclo de Fertilización In Vitro a la que se realizó 24 horas antes la punción folicular bajo
control ultrasonográfico sin incidencias. Acude a Urgencias por presentar dolor pélvico
severo, postración, mareo y aumento del perímetro abdominal. El USG ginecológico es
el que se muestra en la imagen. Señale la afirmación CORRECTA acerca del cuadro que
presenta la paciente:

Se trata de una estimulación ovárica normal: reposo, analgesia, control diuresis y


1. 1.
transferencia embrionaria en 48 horas.

 
 
 

Se trata de un síndrome de hiperestimulación ovárica: reposo, analgesia y


2. 2.
transferencia embrionaria en 48 horas.
Se trata de un síndrome de hiperestimulación ovárica: reposo, analgesia, control
3. 3.
de diuresis y cancelación transferencia embrionaria.
Se trata de un síndrome de hiperestimulación ovárica: laparotomía exploradora y
4. 4.
punción folicular.
Gráfico de respuestas
Comentario

El síndrome de hiperestimulación ovárica es una posible complicación de la fecundación in vitro.


Se trata de una respuesta excesiva del ovario a la inducción de la ovulación, que desencadena una
extravasación de líquido. Las pacientes con síndrome de ovario poliquístico tienen más riesgo de
hiperestimulación. Suele ser un cuadro clínico que se desencadena tras la administración de hCG
y se resuelve en 1- 2 semanas, cursando con distensión abdominal, grandes ovarios llenos de
folículos, dolor abdominal, vómitos, diarrea y ascitis. Puede llegar a ser grave y provocar fracaso
renal, tromboembolismos, SDRA y poner en riesgo la vida de la paciente.

En los casos leves-moderados, el tratamiento consiste en reposo, hidratación y control de las


constantes vitales. En los casos graves puede llegar a requerir una laparotomía urgente, en la que
intentaremos ser lo más conservadores posible. Se debe evitar la gestación cancelando las
inseminaciones o criopreservando los embriones en el caso de pacientes sometidas a fecundación
in vitro.(R/3)

67. En el estudio de esterilidad de la paciente del caso clínico anterior indique cuál de las
siguientes pruebas NO se considera fundamental:

1. 1. Determinación de FSH, LH y estradiol en fase folicular.


2. 2. Ultrasonido ginecológico transvaginal.
3. 3. Histerosalpingografía.
4. 4. Biopsia de endometrio en fase lútea.
Gráfico de respuestas
Comentario

En todo estudio de esterilidad, se deben realizar las pruebas siguientes:

- Anamnesis, exploración y estudio de sangre (BH, bioquímica, orina, Rh y serologías frente a


rubéola, toxoplasma, sífilis, VHB, VHC y VIH).

- USG transvaginal, que nos informa sobre posibles alteraciones morfológicas del útero, anexos,
patología endometrial, endometriosis, ovarios poliquísticos, etc.

- Valoración de la ovulación, midiendo FSH, LH y estradiol en fase proliferativa (3-5 días del ciclo).
Entre el día 20-22 (fase lútea), se medirían PRL y progesterona.

- Seminograma y recuperación de espermatozoides móviles.

- Histerosalpingografía.

Existen otras pruebas que se realizan de forma puntual, cuando existen determinadas sospechas.
Concretamente, la biopsia de endometrio se utiliza cada vez menos y en este caso no existen
razones para realizarla, al menos no de entrada.(R/4)

 
 
 

68. Señale la afirmación ERRÓNEA en relación con el cáncer de vulva:

1. 1. Son factores de riesgo el tabaquismo y la infección por VPH.


2. 2. La localización más frecuente es en labios mayores.
3. 3. El síntoma principal es la hemorragia.
4. 4. La edad de aparición más frecuente es alrededor de los 70 años.
Gráfico de respuestas
Comentario

El síntoma principal del cáncer de vulva NO es la hemorragia, sino el prurito vulvar que tiene una
evolución larga. Generalmente ante cuadros de prurito vulvar de larga evolución, generalmente en
los labios mayores, en mujeres mayores, debemos sospechar cáncer de vulva.(R/3)

69. En pacientes con episodio de metrorragia disfuncional severa, la conducta


terapéutica inmediata es:

1. 1. Histerectomía.
2. 2. Legrado uterino.
3. 3. Administración de oxitócicos por la vía IV.
4. 4. Transfusión sanguínea total fresca.
Gráfico de respuestas
Comentario

La clave en esta prgunta es una metrorragia disfunciónal SEVERA, la conducta a seguir sería el
legrado uterino, respuesta 2, ya que de entrada realizar histerectomía es un tratamiento invasivo
que podríamos utilizarlo en caso que no funcione el legrado.(R2)

70. If a patient with vasculitis presents with saddle nose deformity, which of the following
options is the most likely diagnosis?

1. 1. Polyarteritis nodosa.
2. 2. Henoch-Schönlein purpura.
3. 3. Wegener's granulomatosis.
4. 4. Allergic granulomatosis.
Gráfico de respuestas
Comentario

La deformidad en silla de montar dentro de las vasculitis es propia de la enfermedad de Wegener,


que produce una destrucción del tabique nasal.

Con esta enfermedad debe realizarse diagnóstico diferencial con otras enfermedades que
producen destrucción de la línea media, como el granuloma de la línea media (proceso que forma
parte del grupo de las enfermedades inmunoproliferativas angiocéntricas), neoplasias de las vías
respiratorias superiores y linfomas extranodales, infecciones por hongos, micobacterias o
actinomicosis y la policondritis recidivante.

La biopsia y los cultivos demuestran el diagnóstico, aunque en algunos casos no se observa


vasculitis, y, en ese caso, el resto de la clínica y analítica debe orientarnos al diagnóstico. En el
resto de vasculitis que dan como opciones posibles no se produce destrucción del tabique ni
afectación típica de la vía respiratoria superior(R/3)

 
 
 

71. A 4-month-boy is brought to his pediatrician due to chronic constipation since birth.
His medical history included meconium plug syndrome at birth, which resolved with
enemas. He has been breastfeeding exclusively. Physical examination reveals that his
height and weight are

1. 1. Pathological rectal biopsy.


2. 2. No stool on rectal examination.
3. 3. Relaxation of the internal anal sphincter.
4. 4. Barium retention in the colon for more than 24 hours after its ingestion.
Gráfico de respuestas
Comentario

Estamos ante una enfermedad de Hirschprung o megacolón agangliónico. El diagnostico se realiza


mediante:

- Radiología simple: distensión importante de asas con ausencia de aire a nivel rectal.

- Enema ópaco: se aprecia cambio brusco en el diámetro del colon entre el segmento (estenosado)
y la porción sana (distensión). En el segmento agangliónico aparecen contracciones en dientes de
sierra. En el colon proximal dilatado, los pliegues transversales son paralelos. Existe también un
retraso en la eliminación del contraste.

-Manometría anorrectal: ausencia de relajación del esfinter anal interno ante un aumento de
presión a ese nivel (lo normal es la relajación del mismo).

- Biopsia: proporciona el diagnostico definitivo. En el segmento afectado se observa una ausencia


de células ganglionares (ausencia de plexo de Meissner y Auerbach) con aumento de la
acetilcolinesterasa y de las terminaciones nerviosas. En la zona previa a la zona dañada, existe
hipertrofia muscular.(R/3)

72. Un niño de 5 años presenta fiebre alta, dificultad respiratoria con estridor, disfagia y
babeo. En la exploración destacan las alteraciones del nivel de conciencia, con fases de
irritabilidad y somnolencia. ¿Qué diagnóstico clínico establecería?:

1. 1. Laringomalacia.
2. 2. Epiglotitis aguda.
3. 3. Laringitis aguda espasmódica.
4. 4. Traqueítis.
Gráfico de respuestas
Comentario

Esta pregunta es sencilla e importante. Presenta un caso clínico típico de epiglotitis. Las
alteraciones en la conciencia es signo de gravedad. Recuerde que es una urgencia en cuyo
manejo lo principal es asegurar la vía área.

Para pensar en laringomalacia nos tendrían que haber hablado de una lactante con estridor desde
el nacimiento que empeora en decúbito.

La clínica de laringitis se caracteriza por la triada estridor inspiratorio, tos metálica y dificultad
respiratoria. al contrario que la LAV.

 
 
 

Ante una laringitis que no mejora y con estridor en ambos tiempos respiratorios sospecharemos
traqueitis.(R/2)

73. El concepto de restricción en el crecimiento intrauterino implica:

1. 1. Recién nacido de peso inferior a 2,500 g.


Recién nacido de bajo peso para su edad gestacional independientemente de la
2. 2.
semana de gestación (debajo percentil 10).
3. 3. RN antes de la 37 semana de gestación.
4. 4. Ninguna respuesta es correcta.
Gráfico de respuestas
Comentario

Pregunta sobre el CIR en la que debemos tener claro el concepto. El CIR basa su diagnóstico en
las exploraciones ecográficas. Es una disminución patológica del ritmo de crecimiento fetal, cuyo
resultado será un feto que no alcanza su potencial de crecimiento (peso por debajo del percentil 10
para su edad gestacional).(R/2)

74. En el manejo del paciente asintomático infectado por el VIH se aconseja la realización
de serología para los siguientes gérmenes, EXCEPTO:

1. 1. Citomegalovirus.
2. 2. Toxoplasma.
3. 3. Criptococo.
4. 4. Virus C de hepatitis.
Gráfico de respuestas
Comentario

Puede plantearse la necesidad de realizar serología frente a CMV, ya que muchas personas son
positivas, con independencia de su infección por VIH… Pero, desde luego, lo que no puede pedir
es una serología que no es útil, como sucede con la criptocócica. Aunque esta prueba existe,
carece de utilidad debido a que la sensibilidad es muy escasa.(R/3)

75. Un paciente intervenido hace unos años por una úlcera duodenal acude a consulta
por la aparición de dolor abdominal. Ante la sospecha de una nueva úlcera se le practica
una endoscopía que muestra úlceras en bulbo duodenal y antro pilórico. La gastrina
sérica del paciente está elevada, y se eleva tras la administración de secretina. ¿Cuál
sería el tratamiento óptimo de este paciente teniendo en cuenta su sospecha
diagnóstica?

1. 1. Antagonistas H2 en dosis altas.


2. 2. Omeprazol.
3. 3. Resección completa del tumor.
4. 4. Gastrectomía total.
Gráfico de respuestas
Comentario

La primera sospecha diagnóstica ante un cuadro clínico con úlceras múltiples e hipergastrinemia
debe ser el síndrome de Zollinger- Ellison (gastrinoma), que además se confirma con la elevación
de la gastrina tras la estimulación con secretina. En relación al tratamiento de elección es siempre

 
 
 

quirúrgico, con extirpación del tumor, si se puede (ya que tiene una gran tendencia a la
diseminación).(R/3)

76. Femenino de 63 años que hace 8 semanas se sometió a un by-pass aortocoronario de


3 vasos. Vuelve a ingresar por fiebre de 38ºC y dolor pleurítico. En la base del pulmón
izquierdo se ausculta una disminución de los sonidos respiratorios. Se realiza una Rx en
decúbito lateral, mostrando un pequeño derrame pleural. Se extrae un líquido
serohemático con pH 7.40, LDH 380 U/l (sangre 90), proteínas 3.8 g/dl (sangre 6.3). El
examen citológico es negativo y la gammagrafía de perfusión es normal. ¿Qué
tratamiento o actitud recomendaría?

1. 1. Hacer angiografía pulmonar.


2. 2. Iniciar tratamiento con prednisona.
3. 3. Iniciar tratamiento con indometacina.
4. 4. Realizar un TC de tórax.
Gráfico de respuestas
Comentario

La presencia de dolor pleurítico, con derrame pleural, fiebre y el antecedente de un by- pass
aortocoronario reciente, debe sugerirnos un síndrome post- pericardiotomía. Después de una
intervención quirúrgica que manipule serosas (pleura y/o pericardio), es posible que éstas se
inflamen de forma reactiva, lo que produciría una pleuritis y/o pericarditis durante las siguientes
semanas. Esta entidad cursa con un derrame pleural tipo exudado, como en esta paciente. Como
tratamiento, basta el uso de un AINE, como la indometacina, y esperar a que se resuelva de forma
espontánea, como sucede en la mayoría de los casos.(R/3)

77. A 7-year-old Somalian child is brought to the pediatrician's office for a routine visit.
He is doing well in school and he is able to read and write. Physical examination shows
an adequate development and growth. He has familiar history of sickle cell disease (both
parents and a sister). Hemoglobin electrophoresis shows: Hemoglobin F 0%, Hemoglobin
S 35%, Hemoglobin A 65%. Which would be the most likely complication associated to
his condition?

1. 1. Hematuria
2. 2. Osteonecrosis
3. 3. Spleen infarction
4. 4. Recurrent pneumonia
Gráfico de respuestas
Comentario
Hematuria. Kidney involvement in sickle cell anemia, which is called sickle cell nephropathy,
presents with renal hyperperfusion, glomerular hypertrophy, and glomerular hyperfiltration. Many
patients eventually develop a glomerulopathy leading to glomerular proteinuria. Loss of kidney
function is secondary to cortical infacts. A high percentage of patients suffer painless hematuria,
probably due to medullary infarcts. The rest of the possible options are less common.(R/1)

78. Cuál de las siguientes aseveraciones NO es correcta respecto a la toxicología de la


heroína:

1.     1.   Se  trata  de  una  sustancia  con  un  gran  poder  adictivo.  


El  síndrome  de  abstinencia  aparece  entre  las  8  y  10  horas  después  de  la  última  
2.     2.  
administración.  

 
 
 

3.     3.   La  clonidina  se  utiliza  en  el  tratamiento  del  síndrome  de  abstinencia.  
4.     4.   El  tratamiento  de  elección  ante  un  caso  de  intoxicación  aguda  es  la  naltrexona.  
Gráfico de respuestas
Comentario

La heroína, es una sustancia muy adictiva sobre todo por vía IV > inhalatoria > oral ( opción 1
correcta).

El síndrome de abstinencia depende de la dosis, el tiempo de consumo, y la vida media.


Concretamente con la heroína aparece a partir de las 8 horas (opción 2 correcta) de la última
dosis, con la metadona, al tener vida media más larga, la abstinencia es más larga pero más leve y
por eso se utiliza como sustitutivo para la deshabituación. El opiode que más pronto induce
abstinencia la meperidina. Para evitar los síntomas de la abstinencia se una clonidina y así
disminuye la hiperactividad adrenérgica con agonistas alfa- 2.

Lo que es falso es que la naltrexona esté indicada para el tratamiento de la intoxicación aguda. Lo
que se usa es la Naloxona! (R/4)

79. Una de las siguientes NO es complicación frecuente del abruptio:

1. 1. Insuficiencia renal.
2. 2. Choque.
3. 3. Síndrome de Sheehan.
4. 4. Pielonefritis.
Gráfico de respuestas
Comentario

Como complicaciones del abruptio, se han descrito todas las respuestas, excepto la pielonefritis
(opción 4). Las más frecuentes son la CID y la insuficiencia renal. La CID también se presenta en
abortos diferidos y la patogenia es similar en ambos casos: liberación y paso de tromboplastinas
fetales a circulación materna. El tratamiento consiste en una adecuada reposición de factores de
coagulación y fibrinógeno, al mismo tiempo procedemos a la evacuación inmediata del útero,
mediante la realización de una cesárea. El choque y el síndrome de Sheehan son complicaciones
menos frecuentes.(R/4)

80. Niño de 4 años con fiebre alta de 6 días de


evolución acompañada de irritabilidad. La madre refiere además haberle notado las
manos y los pies hinchados y dolorosos. A la exploración llama la atención hiperemia

 
 
 

conjuntival intensa sin exudado, un exantema de predominio en tronco y labios


enrojecidos y resecos. Es ingresado con sospecha de enfermedad infecciosa y tratado
con antibióticos tras la toma de cultivos. Al 10º día de evolución, la fiebre no ha
desaparecido y comienza a presentar alteraciones en manos y pies como los que se
muestran en la imagen. En relación al cuadro clínico citado, señale la opción FALSA:

La imagen muestra la descamación en dedo de guante característica de la


1. 1.
enfermedad.
Para confirmar el diagnóstico es necesario realizar biopsia de algún ganglio
2. 2.
linfático accesible.
3. 3. La base patogénica de la enfermedad es una vasculitis.
La etiología de la enfermedad no está clara pero se basa en la teoría del
4. 4.
superantígeno.
Gráfico de respuestas
Comentario

La imagen, en esta pregunta, es sólo un dato más, y nos muestra lo que ya nos podríamos haber
imaginado por el contexto de la pregunta: descamación plantar. Se trata de una enfermedad de
Kawasaki, en cuya patogenia parece estar implicado un superantígeno que produciría una
sobreactivación policlonal de los linfocitos T, que producirían citoquinas responsables de las
manifestaciones de la enfermedad.
Recuerden que, en esta patología, el problema fundamental es que a veces se producen
dilataciones aneurismáticas en las coronarias, por lo que estos niños estarían predispuestos a
infartos de miocardio a edades muy precoces. El diagnóstico de la enfermedad de Kawasaki se
realiza en base a criterios clínicos y no precisaría confirmación histológica (R/2).

81. El niño de la pregunta anterior fue ingresado y tratado con gammaglobulina


intravenosa además de aspirina. Sobre el manejo de este paciente, señale la opción
FALSA:

Si la fiebre no cede con gammaglobulina el fármaco de segunda elección son los


1. 1.
corticoides.
La gammaglobulina es eficaz en la prevención de la aparición de aneurismas
2. 2.
coronarios en la mayoría de los casos.
La aspirina a dosis antiagregantes es importante en la prevención de las
3. 3.
trombosis.
La descamación en dedo de guante es característica de la fase aguda de la
4. 4. enfermedad y por tanto muy útil para el diagnóstico precoz y el inicio del
tratamiento.
Gráfico de respuestas
Comentario

La descamación es habitual cuando la enfermedad ya lleva cierta evolución, por lo que no tendrá
aplicación alguna para llegar a un diagnóstico precoz (respuesta 4 falsa). Tal como explicábamos
en el comentario anterior, el problema principal que puede complicar este cuadro son los
aneurismas coronarios, que aparecen con menor frecuencia cuando se utiliza la gammaglobulina
intravenosa, como se comenta en la opción 2.(R/4)

82. El fenómeno de Koebner se da en:

 
 
 

1. 1. Pénfigo.
2. 2. Psoriasis.
3. 3. Púrpura trombocitopénica.
4. 4. Vasculitis por hipersensibilidad.
Gráfico de respuestas
Comentario

Pregunta básica sobre un signo muy útil en el diagnóstico dermatológico, el fenómeno de Koebner.
Consiste en la aparición de determinadas dermatosis en las zonas de presión o traumátismo. Es
típico de las enfermedades eritemato-escamosas como psoriasis (opción 3), liquen plano, pitiriasis
rubra pilaris y Darier. Infecciosas con el molluscum o las verrugas y de otras como vitíligo,
xantomas, vasculítis y síndrome de Sweet. La consecuencia práctica es que el paciente debe
conocer que el fenómeno puede actuar como factor precipitante, por lo que debe evitar en la
medida de lo posible las lesiones cutáneas.(R/2)

83. El tratamiento más recomendable de la diverticulosis asintomática es:

1. 1. Laxantes salinos.
2. 2. Anticolinérgicos.
3. 3. Resección quirúrgica del segmento afecto.
4. 4. Dieta rica en residuos.
Gráfico de respuestas
Comentario

Cuando se habla de diverticulosis, el término se refiere a la presencia de divertículos, sin


complicación acompañante. La diverticulosis aumenta en frecuencia con la edad, y suele ser
asintomática. Por tanto, es evidente que, de inicio, no se indica ningún tratamiento quirúrgico. En
situación estable, sin clínica, no se precisa el uso de fármacos de ningún tipo. En cambio, sí parece
aconsejable una dieta rica en fibra y residuos, que “arrastre” el contenido del colon, lo que dificulta
el estancamiento de restos fecales intradiverticulares, lo que incrementa el riesgo de
diverticulitis.(R/4)

84. A patient with a past history of HIV infection comes to the emergency department
complaining of dyspnea and pallor. Current medication includes antiretroviral therapy
with AZT + 3TC + Efavirenz, for 14 months. Last determinations of CD4 and viral load were
350/mm3 and <200 copies / ml, respectively. Current laboratory studies show:
hemoglobin 7.8 g / dl, MCV 68 fl. The most likely diagnosis is:

1. 1. Aplastic crisis (parvovirus B19 infection).


2. 2. Disseminated Leishmaniasis.
3. 3. Disseminated infection by Mycobacterium avium-intracellulare.
4. 4. Chronic gastrointestinal bleeding.
Gráfico de respuestas
Comentario

Pregunta sencilla. Tenemos que percatarnos que se trata de un paciente con diagnóstico de VIH
controlado, con unos CD4 de 350 y una carga viral <100 copias, por lo que al encontrarnos con
una anemia microcítica, lo primero que tendriamos que descartar es una hemorragia
gastrointestinal crónica.(R/4)

 
 
 

85. A 79-year-old male is brought to the ER due to sudden onset intense abdominal pain,
with vomiting. His past medical history is positive for hypertension, diabetes, atrial
fibrillation and two ischemic strokes. His BP is 150/98 mm Hg, pulse is 120/min,
temperature is 38.4ºC and respirations are 20/min. Physical examination reveals clear
lungs to auscultation, severe abdominal pain to palpation and absent bowel sounds, with
rebound. Rectal examination shows heme-positive stools. Lab tests show: WBC
18,700/mm3, hemoglobin 13 g/dl, creatinine 1 mg/dl, troponin I < 0.017 (normal) INR 1.5.
What is the most likely diagnosis?

1. 1. Acute pancreatitis
2. 2. Acute diverticulitis
3. 3. Myocardial infarction
4. 4. Bowel infarction
Gráfico de respuestas
Comentario
Bowel infarction. Acute mesenteric ischemia (AMI) is a syndrome caused by inadequate blood flow
through the mesenteric vessels, resulting in ischemia and eventual gangrene of the bowel wall.
Although relatively rare, it is a potentially life-threatening condition. Broadly, AMI may be classified
as either arterial or venous. AMI as arterial disease may be subdivided into nonocclusive
mesenteric ischemia (NOMI) and occlusive mesenteric arterial ischemia (OMAI); OMAI may be
further subdivided into acute mesenteric arterial embolism (AMAE) and acute mesenteric arterial
thrombosis (AMAT). In this case, the patient presents wth occlusive mesenteric arterial ischemia,
most likely due to embolism. Of all the types of AMI, acute mesenteric arterial embolism typically
has the most abrupt and painful presentation as a consequence of the rapid onset of occlusion and
the inability to form additional collateral circulation. It has been described as abdominal apoplexy
and is sometimes referred to as a “bowel attack.” Often, vomiting and diarrhea (gut emptying) are
observed. Patients are usually found to have some cause of emboli. Because most emboli are of
cardiac origin, patients often have atrial fibrillation or a recent myocardial infarction (MI) with mural
thrombus. Early in the course of AMI, in the absence of peritonitis, physical signs are few and
nonspecific. Peritoneal signs develop late, when infarction with necrosis or perforation occurs.
Fever, hypotension, tachycardia, tachypnea, and altered mental status are observed.(R/4)

86. En relación al eczema marginado de hebra señale la respuesta INCORRECTA:

1. 1. Clínicamente puede aparecer un borde inflamatorio, incluso con pústulas.


2. 2. Son características unas pequeñas lesiones satélites.
3. 3. Ceden bien al tratamiento con imidazoles tópicos.
4. 4. Puede transmitirse por contacto sexual.
Gráfico de respuestas
Comentario

El eczema marginado de Hebra se trata de una tiña localizada a nivel inguinal, que se produce por
hongos dermatofíticos, concretamente por el E. flocosum y T. rubrum (especies también
relacionadas con la tiña pedis). El contagio se produce por contacto directo o a través de toallas,
ropas...Entre los síntomas que se producen destaca el prurito. Las lesiones comienzan por una o
varias manchas eritematosas, ligeramente descamativas, que progresan de forma excéntrica y dan
lugar a una placa más grande, con borde de crecimiento activo eritematoescamoso y en ocasiones
con pequeñas vesículas o pústulas produciendo aclaramiento central. Puede aparecer alguna
lesión satélite pequeña, pero no es característico.(R/2)

87. Todos los siguientes datos apoyan el diagnóstico de apraxia constructiva, EXCEPTO:

 
 
 

El paciente no es capaz de copiar en un papel un dibujo modelo cuando se le pide


1. 1.
verbalmente.
2. 2. El paciente comprende lo que se le está pidiendo.
3. 3. El paciente es arquitecto de profesión.
4. 4. El paciente presenta una hipoestesia hemicorporal derecha.
Gráfico de respuestas
Comentario

Esta pregunta sirve para repasar el tema de las apraxias. Debe recordar que la apraxia es la
incapacidad para llevar a cabo actos motores ante una orden verbal o imitación en un paciente con
una adecuada comprensión y sin déficits motores o sensitivos primarios. La apraxia constructiva,
más concretamente, es la incapacidad para dibujar o construir figuras o formas bi o
tridiminesionales. Se explora pidiendo al paciente que copie en un papel varios dibujos
modelo.(R/4)

88. A 2-year-old child is brought to the emergency department by her worried mother
presenting with fever and crying while urinating. The girl had previously been healthy and
had a normal development. The mother has also observed some blood in the urine
collected. Rectal temperature is 37.8ºC. Vital signs are: 100/70 mmHg (blood pressure)
and 99 bpm. Abdominal palpation is painful in suprapubic and right costovertebral areas.
Urine test results are: Gravity 1.004, pH 7, None proteins, Moderate blood, Negative
glucose, Negative ketones, Leukocyte esterase positive, positive nitrates, White blood
cells 45 +/hpf, Red blood cells 20-30/hpf, Casts none, Crystals none. Once antibiotherapy
is completed with good course, what is the best following step in the management of this
patient?

1. 1. Chronic prophylactic antibiotherapy


2. 2. Bladder and renal ultrasonography and voiding cystourethrogram.
3. 3. Renal and bladder ultrasound
4. 4. Expectation with observation
Gráfico de respuestas
Comentario
Bladder and renal ultrasonography and voiding cystourethrogram. Urinary tract infections are
common in children, especially girls and boys under one year of age. If infections appear at later
ages, it should raise the suspicion of renal abnormalities or urinary tract defects. Ultrasound is the
diagnostic test of choice in the acute phase because it can help to diagnose abscesses in the renal
parenchyma or obstruction in the urinary tract. It is an inexpensive, noninvasive test with no
radiation exposure.(R/2)

89. ¿Cuál de los siguientes antihipertensivos NO disminuye los niveles de renina?:

1. 1. Metildopa.
2. 2. Reserpina.
3. 3. Propranolol.
4. 4. Diazóxido.
Gráfico de respuestas
Comentario

Es importante que domines el tratamiento farmacológico de la HTA porque es muy preguntado en el ENARM
pero no se preocupe si ha fallado esta pregunta porque es difícil. Para acertarla, debe tener clara la fisiología
del sistema renina- angiotensina- aldosterona. Tenga en cuenta que la renina se libera en respuesta a varios

 
 
 

estímulos, entre los cuales se encuentran la activación del SNS y de las células yuxtaglomerulares. Fíje que
en las dos primeras opciones aparecen fármacos que bloquean este sistema, disminuyendo así la secreción
de renina. La metildopa y la clonidina son fármacos simpaticolíticos de acción central y la reserpina es un
depletor de catecolaminas. El propranolol es un betabloqueante que bloquea los receptores beta- 1 de las
células yuxtaglomerulares, inhibiendo así la liberación de renina. El diazóxido, sin embargo, es un
vasodilatador de uso intravenoso que disminuye la tensión arterial por otro mecanismo distinto.(R/4)

90. ¿Cuál de las siguientes manifestaciones NO es característica de una colestasis


prolongada?:

1. 1. Prurito.
2. 2. Osteopenia.
3. 3. Degeneración neuromuscular.
4. 4. Estreñimiento.
Gráfico de respuestas
Comentario
En el síndrome colestático, parte de la sintomatología se debe al déficit de vitaminas liposolubles.
Así, puede surgir osteopenia u osteomalacia por alteraciones en la absorción proteica y de la
vitamina D; y alteraciones neuromusculares o ataxia si hay déficit de vitamina E, situación de baja
frecuencia y que se observa básicamente en niños. Por otra parte, hay síntomas derivados de la
retención de bilis como ictericia, hipercolesterolemia, prurito y en ocasiones puede encontrarse
diarrea tipo esteatorrea por falta de absorción de las grasas al no aportarse sales biliares. No se ha
descrito estreñimiento.(R/4)

91. En la hemocromatosis idiopática primaria NO es característico encontrar:

1. 1. Disminución de la ferritina sérica.


2. 2. Elevación de la sideremia.
3. 3. Hiperglucemia.
4. 4. Saturación de la transferrina mayor del 70%.
Gráfico de respuestas
Comentario

Recuerda que un hallazgo característico de la hemocromatosis primaria es la elevación de la


ferritina, por aumento de los depósitos de hierro.(R/1)

92. La escarlatina es producida por:

1. 1. Adenovirus.
2. 2. Rotavirus.
3. 3. Neumococo.
4. 4. Toxina eritrogenica del estreptococo B hemolítico.
Gráfico de respuestas
Comentario

La escarlatina está ocasionada por S. pyogenes productor de toxinas eritrógenas. Afecta


principalmente a escolares entre 5-15 años y es infrecuente en menores de 3 años.

 
 
 

El tratamiento de elección es penicilina oral durante 10 dias.(R/4)

93. ¿Cuál de los siguientes NO es un factor protector para las siguientes patologías que
se mencionan a continuación?:

1. 1. Multiparidad en el cáncer de endometrio.


2. 2. Nuligesta en cáncer de cérvix.
3. 3. Anovulatorios en cáncer de ovario.
4. 4. Anovulatorios en la enfermedad inflamatoria pélvica.
Gráfico de respuestas
Comentario

El tema importante sobre de los factores de riesgo y protección en ginecología es importante,


sobre todo de los tumores.

El principal factor de riesgo para el cáncer de ovario es la ovulación incesante por lo que la
anovulación (respuesta 3) es un factor protector.

Los anticonceptivos orales además de proteger de el cancer de ovario, lo hacen del


adenocarcinoma de endometrio. En cuanto al cérvix, aumentan la displasia pero NO el cancer. Y le
pude ayudar recordar que disminuyen la EIP pero aumentan la vulvovaginitis.

El principal factor de riesgo en el cáncer de endometrio es la estimulación estrogénica sin


oposición de progesterona. Son factores de protección: menarquia tardia, multiparidad,
menopausia precoz...

Las nuligestas tienen mas riesgo de cáncer de endometrio y de ovario pero no necesariamente de
cérvix ya que el principal el principal factor de riego para cervix es el numero y edad de inicio de las
relaciones sexuales. El que sea nuligesta no implica que no tenga relaciones sexuales, por lo que
no puede ser considerado como factor de protección.(R/2)
on verdaderas.
Gráfico de respuestas
Comentario

El neuroblastoma es el tumor sólido extracraneal más frecuente en la infancia. La edad media de


los pacientes en el momento del diagnóstico son los dos años (el 90 % de los casos se producen
en menores de cinco años).

La localización más frecuente es el abdomen 70%. Dentro del abdomen, un 50 % se econtrarán en


suprarrenales. Otras localizaciones: tórax, mediastino, posterior y nasofaringe.

Se asocia a síndrome alcohólico fetal, hidantoínas, neurofibromatosis tipo 1, nesidioblastosis,


Hirschprung.

Clínica deriva de la localización, puede presentar algunos síndromes complejos como: síndrome de
opsoclono-mioclono, diarrea secretora intensa, hipertensión arterial y hematoma lineal en el
párpado.

Los estadios según International Neuroblastoma Staging System (INSS) son los siguientes:

 
 
 

I: localizado en el órgano de origen .

II: situado más allá de la estructura de origen, sin sobrepasar la línea media y sin afectación
ganglionar ipsilateral (HA) o con ella (IIB).

III: más allá de la línea media con/sin afectación ganglionar.

IV: muestra metástasis al hueso, a la médula ósea, al hígado, a la piel , a los ganglios linfáticos
distantes o en otras localizaciones .

IVs: tumor estadio I o II en un menor de un año con metástasis en el hígado, la piel o la médula
ósea.(R/4)

94. Un militar de la brigada paracaidista, de 25 años de edad, sufre un traumatismo


durante un entrenamiento. Como consecuencia de éste, sufre una fractura vertebral en
C6, así como una fractura de Colles en antebrazo izquierdo. Se recupera y, semanas
después, le cuenta que persiste cierto dolor, limitación de la movilidad y hormigueo en
los tres primeros dedos de la mano izquierda, que empeora claramente durante la noche.
Teniendo en cuenta el diagnóstico más probable, señale cuál de las siguientes causas
NO predispone al mismo:

1. 1. Desviaciones óseas postraumáticas.


2. 2. Amiloidosis.
3. 3. Artritis reumatoide.
4. 4. Hipertiroidismo.
Gráfico de respuestas
Comentario

Nos están describiendo una secuela de la fractura de Colles como es un síndrome del túnel del
carpo. Éste puede ser debido a muchas patologías más, todas las descritas en la pregunta excepto
el hipertiroidismo (sin embargo, sí se asocia al hipotiroidismo).(R/4)

95. Respecto al síndrome hepatopulmonar, ¿cuál de las siguientes afirmaciones es


FALSA?

1. 1. Es un cuadro clínico de carácter básicamente funcional.


2. 2. Existe un desequilibrio de las relaciones ventilación/perfusión pulmonares.
Es el causante de la hipoxemia arterial que presentan algunos pacientes con
3. 3.
hepatopatía crónica.
4. 4. Es irreversible incluso después del trasplante hepático.
Gráfico de respuestas
Comentario

El síndrome hepatopulmonar es un cuadro que se produce en pacientes con hepatopatia crónica


(igual que el hepatorrenal), y cursa con hipoxemia arterial debida en parte a la menor reactividad
vascular pulmonar a estímulos vasoconstrictores pulmonares (como la hipoxia). No obstante, al ser
una patología funcional, es reversible si se realiza un trasplante hepático con éxito.(R/4)

96. Los pliegues urogenitales dan lugar a:

 
 
 

1. 1. Clítoris.
2. 2. 2/3 inferiores de la vagina.
3. 3. Labios menores.
4. 4. Vestíbulo.
Gráfico de respuestas
Comentario

Pregunta muy difícil de Embriología.

Genitales externos: Durante la quinta semana de vida embrionaria, se forman pliegues tisulares a
cada lado de la cloaca que se unen por delante en la linea media para formar el tubérculo genital.
Con la división de la cloaca por el tabique urorrectal y la formación consecuente del perineo, estos
pliegues cloacales se conocen por delante como pliegues urogenitales y por detrás como pliegues
anales.

El tubérculo genital empieza a aumentar de tamaño, pero en el embrión del sexo femenino, su
crecimiento va haciendose mas lento de manera gradual hasta que se convierte en el clítoris, y los
pliegues urogenitales forman los labios menores.

En el varon, el tubérculo genital sigue creciendo para formar el pene, y se cree que los pliegues
urogenitales se fusionan para encerrar a la uretra del pene. Lateralmente a los pliegues
urogenitales, se desarrolla otro par de eminencias, que se conocen en la etapa indiferente como
eminencias labioscrotales.

En ausencia de andrógenos se quedan prácticamente sin fusionar y se convierten en labios


mayores.

El seno urogenital definitivo da origen al vestíbulo vaginal, en el cual se abren uretra, vagina y
glandulas vestibulares mayores. (R/3)

97. Señale la afirmación ERRÓNEA sobre el oncogén C-erb B2 (Her-2-neu):

1. 1. También se llama receptor del factor de crecimiento epidérmico humano 2.


Las pacientes con cánceres que lo sobreexpresan pueden ser tratadas con
2. 2.
trastuzumab.
3. 3. Los cánceres de mama en los que se sobreexpresa tienen mejor pronóstico.
Se ha relacionado con resistencia al tratamiento antihormonal, pero puede hacer
4. 4.
que sea más sensible a ciertos quimioterápicos.
Gráfico de respuestas
Comentario

Esta pregunta puede confundirnos si no dominamos este concepto. Los cánceres de mama que
sobreexpresan el Her- 2- neu (C- erb B2) generalmente son más agresivos y suelen tener
receptores hormonales negativos. Por tanto NO tienen mejor pronóstico como se afirma en la
opción 3. Otra cosa es que tienen un tratamiento específico con Trastuzumab (Herceptin®) y es lo
que nos hace pensar, erróneamente, que tienen mejor pronóstico.(R/3)

98. Femenino con diagnóstico reciente de adenocarcinoma de recto que se encuentra a


10 cm del margen anal. ¿Cómo se realiza el estudio de extensión?

1. 1. TC abdominal.

 
 
 

2. 2. Resonancia magnética.
3. 3. Rx simple de abdomen.
4. 4. TC torácico.
Gráfico de respuestas
Comentario

Ante un carcioma colorrectal, debemos hacer un estudio de extensión para valorar la N y la M del
TNM. La prueba de imagen de elección para conseguir éste objetivo es el TC. Respuesta 1
correcta.(R/1)

99. A 40-year-old man comes to the ER complaining of a 4-day history of abdominal pain
and bloody diarrhea. He reports 6 similar episodes in the past year. Endoscopic
evaluation shows extensive disease from terminal ileum to the rectum, with ulcerations
and pseudopopyps. About the disease that you suspect, which of the following is
pathognomonic?

1. 1. Discontinuous bowel involvement.


2. 2. Involvement of terminal ileum.
3. 3. Noncaseating granulomas on biopsy.
4. 4. Crypt abscesses on biopsy.
Gráfico de respuestas
Comentario
Crypt abscesses on biopsy. Our main suspition is ulcerative colitis, which typically presents cryps
abscesses. Noncaseating granulomas and cobblestone appearance in endoscopy are charateristic
of Crohn's disease(R/4)

100. Recién nacido de 3 días de vida


cuyos padres traen a urgencias por presentar vómitos biliares desde el nacimiento junto
con evacuaciones escasas, acólicas, sin productos patológicos. Está siendo alimentado

 
 
 

con lactancia materna exclusiva, presenta buena succión y realiza bien las tomas. Los
padres refieren además tinte ictérico desde las 48 horas de vida. No presenta fiebre ni
otra sintomatología. A su llegada a urgencias presenta los siguientes signos vitales: FC.
130 l.p.m. FR. 37 r.p.m, Tª 36.8 ºC, TA. 61/30 mmHg, saturación de O2 96 %. En la
exploración física del paciente se aprecia un adecuado estado de hidratación, con
ictericia cutánea hasta miembros inferiores, con auscultación cardiopulmonar normal y
un abdomen blando y depresible en el que no se palpan masas ni visceromegalias. En la
exploración neurológica se aprecia una fontanela anterior normotensa, se encuentra
reactiva, con buen tono muscular y reflejos arcaicos presentes y simétricos. Se realizan
examenes de laboratorio en la que se detecta hiperbilirrubinemia a expensas de
bilirrubina indirecta. Asimismo, se lleva a cabo un ultrasonido abdominal y
posteriormente un tránsito gastroduodenal, que puede ver en la imagen. Ante la historia
clínica, la exploración física y las pruebas complementarias, ¿cuál es el diagnóstico más
probable?

1. 1. Atresia duodenal.
2. 2. Atresia esofágica.
3. 3. Atresia pilórica.
4. 4. Atresia anal.
Gráfico de respuestas
Comentario

El paciente de esta pregunta presenta una obstrucción intestinal. La presencia de vómitos de


características biliares descarta una estenosis proximal al duodeno, ya que la vesícula biliar
desemboca en la primera porción del duodeno, por ello podemos descartar las respuestas 2, 3 y 4.

La estenosis hipertrófica de píloro no ocurre en los primeros días de vida, sino entre las 3 y las 6
semanas de vida. Se caracteriza por vómitos progresivos, alimenticios, nunca biliares.

La atresia pilórica presenta el mismo cuadro pero desde el nacimiento. Recuerde también que la
atresia esofágica más frecuente es la tipo tres (aTRESia esofágica)

La atresia anal se manifiesta por ausencia de evacuaciones y se detectaría en la exploración física.

El diagnóstico en este caso, es una atresia duodenal por los vómitos biliares, por el inicio neonatal
y por la presencia de dos burbujas en el tránsito (también visible en radiografía simple), que
corresponden una al estómago y otra al duodeno, sin que pase el contraste distalmente a la
estenosis.(R/1)

101. El paciente de la pregunta anterior fue sometido a intervención quirúrgica con una
evolución favorable. Respecto al cuadro clínico que presenta, ¿cuál de las siguientes
afirmaciones es CORRECTA?

1. 1. Se asocia a otras enfermedades, como el síndrome de Down.


2. 2. En la radiografía de abdomen se observa el signo de la burbuja.
3. 3. El tratamiento quirúrgico de elección es la pilorotomía extramucosa de Ramsted.
Aunque puede presentarse en el período neonatal, es típico que debute en la
4. 4.
tercera semana de vida.
Gráfico de respuestas
Comentario
 
 
 

El paciente presenta una atresia duodenal, cuyo principal diagnóstico diferencial es la obstrucción
a nivel del píloro.

La atresia duodenal debuta en los primeros días de vida, ya que el niño nace con una estenosis a
ese nivel, a diferencia de lo que ocurre en la estenosis hipertrófica de píloro en la que esa
obstrucción es progresiva, por lo que se manifiesta hasta la 3ª y la 6ª semanas de vida.

En la radiografía de la atresia duodenal observaremos 2 burbujas, una gástrica y otra duodenal,


mientras que en la obstrucción a nivel del píloro sólo veremos una burbuja, la correspondiente al
estómago.

La estenosis hipertrófica de píloro se asocia a la toma de macrólidos durante la lactancia y el


embarazo; sin embargo, la atresia duodenal se relaciona con el síndrome de Down (recuerde las 3
D: atresia Duodenal, síndrome de Down y Doble burbuja).

La pilorotomía extramucosa de Ramsted es el tratamiento de elección para la estenosis pilórica, no


para la atresia duodenal.(R/1)

102. Todo lo siguiente puede aparecer como consecuencia de la resección ileal,


EXCEPTO:

1. 1. Déficit de hierro.
2. 2. Esteatorrea.
3. 3. Aumento de la frecuencia de cálculos de colesterol.
4. 4. Diarrea por pérdida de la función de la válvula ileocecal.
Gráfico de respuestas
Comentario

La resección ileal no asocia déficit de hierro, pues este elemento se absorbe fundamentalmente en
regiones proximales del tracto gastrointestinal, sobre todo en el duodeno y el yeyuno proximal.
Recuerda que la absorción es mayor en forma hem, luego en forma ferrosa y como peor se
absorbe es en forma férrica.(R/1)

103. En relación al ciclo genital femenino es FALSO que:

1. 1. En la primera fase del ciclo, la FSH produce un aumento del recuento folicular.
2. 2. La LH tiene un solo pico, previo a la ovulación.
3. 3. El cuerpo lúteo produce progesterona y estrógenos.
4. 4. Al final de la fase lútea se produce un descenso de FSH previo a la menstruación.
Gráfico de respuestas
Comentario
Pregunta que versa sobre los conocimientos básicos del ciclo genital femenino. La GnRH
sintetizada en el hipotálamo estimula la secreción de FSH y LH por la hipófisis. La FSH tiene un
pico en primera fase (folicular) para estimular una cohorte folicular, y al final de la fase lútea
también se eleva sus niveles para estimular un nuevo grupo de folículos.(R/4)

104. ¿Cuál de los procesos cutáneos siguientes es MENOS frecuente en infectados por
el virus de la inmunodeficiencia humana?:

 
 
 

1. 1. Foliculitis eosinofílica.
2. 2. Exantema agudo retroviral.
3. 3. Reacción cutánea a medicamentos.
4. 4. Xerodermia.
Gráfico de respuestas
Comentario
Pregunta de alta dificultad, pues para contestarla, es preciso memorizar toda una lista de
afectaciones dermatológicas relacionadas con la infección por el VIH.(R/1)

105. Uno de los síntomas siguientes NO es característico de la anorexia nerviosa:

1. 1. Angustia.
2. 2. Perturbación de la imagen corporal.
3. 3. Interés por vestir ropas amplias.
4. 4. Amenorrea.
Gráfico de respuestas
Comentario

Es una pregunta muy sencilla pero hace dudar sobre las características clínicas básicas dela
anorexia nerviosa. Todas las opciones excepto la 3 se observan en la anorexia. Más que interés en
vestir ropas amplias, se esconden en ella para ocultar su delgadez. Las anoréxicas están además
en alerta y generalmente tristes, y apartadas aunque dependen de la fase. Generalmente son
hiperactivas aunque al final se convierten en hipo. Tienen cambios bruscos del estado de ánimo,
con rigidez de pensamiento, controladoras, manipuladoras, son desconfiadas, luchan por la
percepción, con personalidad obsesivo- compulsiva. No se olvide de repasar los criterios
diagnósticos.(R/3)

106. ¿Cuál de las siguientes aseveraciones en la fibrosis pulmonar idiopática es FALSA?:

1. 1. Fisiopatológicamente no se diferencia del conjunto de las fibrosis pulmonares.


Existen enfermos que evolucionan muy rápidamente (Hammam-Rich), pero la
2. 2.
mayoría tienen una evolución más lenta.
3. 3. En el lavado bronquialveolar, el hallazgo más común es la neutrofilia.
4. 4. Los corticoides son efectivos tanto en la fase de alveolitis como en la de fibrosis.
Gráfico de respuestas
Comentario
El tratamiento de la fibrosis pulmonar son los corticoides cuyo efecto es disminuir o suprimir la
alveolitis puesto que la fibrosis es irreversible. Cuando no hay respuesta se pueden anadir
inmunosupresores como la ciclofosfamida o azatioprina. Sólo hay respuesta en el 20% de los
casos. El trasplante de pulmón debe considerarse en la fases finales de la enfermedad.(R/4)

107. Todas las afirmaciones sobre la enterocolitis necrosante (ECN) son ciertas,
EXCEPTO:

Su incidencia es del 1 al 5 por 100 de los ingresos de recién nacidos en unidades


1. 1.
de cuidados intensivos.
2. 2. La situación de hipoxia no favorece este proceso.
3. 3. El estrés perinatal es un factor predisponente.
4. 4. El diagnóstico depende de mantener una sospecha constante sobre este proceso.
Gráfico de respuestas

 
 
 

Comentario

La enterocolitis necrotizante afecta fundamentalmente a íleon distal y colon proximal. Suele


relacionarse con situaciones de hipoxia y bajo gasto (respuesta 2 correcta). Clínicamente, debe
sospecharla ante la presencia de distensión abdominal y evacuaciones sanguinolentas. Existen
varios signos radiológicos, pero el más sugestivo es laneumatosis intestinal, que es diagnóstico de
esta enfermedad. El tratamiento consiste en dieta absoluta, sonda nasogástrica e hidratación
intravenosa. Debe administrase además antibioterapia, cubriendo gérmenes anaerobios y
gramnegativos. En caso de perforación intestinal o refractariedad a antibióticos, se plantearía la
cirugía.(R/2)

108. Señale la FALSA respecto a los tumores benignos del pulmón:

Los hamartomas, lo más frecuente es que sean asintomáticos y de localización


1. 1. periférica, y a pesar de su benignidad, se suelen resecar para asegurar su
benignidad.
La histología más frecuente de los tumores benignos del pulmón es adenoma
2. 2.
bronquial.
Lo más habitual en el tumor carcinoide del pulmón es que se diagnostique por el
3. 3. síndrome carcinoide, que además puede producirse sin necesidad de que existan
metástasis hepáticas.
4. 4. El tratamiento de elección de todos los adenomas es la resección quirúrgica.
Gráfico de respuestas
Comentario

La respuesta incorrecta es la 3, pues el tumor carcinoide pulmonar se diagnostica con mayor


frecuencia por una tos persistente, debido a su crecimiento endobronquial. La aparición de
síndrome carcinoide se relaciona con tumores de cierto tamaño, con lo que aparece después.
Recuerde que el tumor carcinoide constituye la histología más frecuente de los tumores benignos
del pulmón y su localización es central, ya que crece en el interior de los bronquios y deriva de las
células del sistema APUD (neuroendocrinas).(R/3)

109. De las siguientes enfermedades, ¿cuál es la que con mayor frecuencia produce
ulceraciones múltiples en las primeras porciones (duodeno-yeyuno) del intestino
delgado?

1. 1. Antinflamatorios no esteroideos (AINE).


2. 2. Enfermedad de Crohn.
3. 3. Tuberculosis intestinal.
4. 4. Infección por Yersinia.
Gráfico de respuestas
Comentario

La localización más corriente de la tuberculosis en el ámbito digestivo es el íleon terminal. En


consecuencia, sería raro pensar en las primeras porciones del intestino delgado, aparte de que la
tuberculosis gastrointestinal no es una enfermedad frecuente (respuesta 3 falsa).

La enfermedad de Crohn afecta preferentemente a las regiones terminales del intestino delgado
(íleon terminal). Recuerda que, en el diagnóstico diferencial de la enfermedad de Crohn, está la
ileítis por Yersinia, puesto que afecta a la misma zona (opciones 2 y 4 falsas).(R/1)

 
 
 

110. ¿Qué paciente está sometida a un mayor riesgo de cáncer ovárico de estirpe
epitelial?

1. 1. Tomadora de anovulatorios durante 10 años.


2. 2. Síndrome de ovario poliquístico.
3. 3. Disgenesia gonadal.
4. 4. Tratamiento con clomifeno o gonadotropinas.
Gráfico de respuestas
Comentario

El 90% de las tumoraciones malignas de ovario son de estirpe epitelial. Los factores de riesgo
tienen como punto en común el traumatismo constante en el recubrimiento epitelial del ovario, con
la posibilidad de que la reparación no sea adecuada y aparezcan células atípicas. Ejemplo claro de
ello: un ovario sometido a sucesivas inducciones de la ovulación, con numerosos folículos
reclutados desarrollándose y rompiéndose en cada ciclo (respuesta 4). Serán factores protectores,
por tanto, el consumo de anticonceptivos hormonales (anovulatorios) y el SOP (dónde con gran
frecuencia se producen ciclos anovulatorios). Aunque el síndrome de Morris predispone a un
cáncer de ovario, no sería de estirpe epitelial, sino germinal (el disgerminoma) y algunas
disgenesias gonadales también (el síndrome de Swyer se asocia con la aparición de
gonadoblastomas).(R/4)

111. Mujer de 59 años que acude a realizarse la mastografía del programa de screening
de cáncer de mama. Se observa un nódulo de 2 cm de diámetro, con bordes espiculados
y con microcalcificaciones agrupadas en su interior. La anatomía patológica de la pieza
quirúrgica confirma que se trata de un cáncer ductal infiltrante, con 3 de los 11 ganglios
extirpados positivos para metástasis. Indique cómo completaría el tratamiento:

1. 1. Radioterapia y poliquimioterapia.
2. 2. Radioterapia y hormonoterapia.
3. 3. Completar la mastectomía y añadir quimioterapia.
4. 4. Completar la mastectomía y añadir quimioterapia y hormonoterapia.
Gráfico de respuestas
Comentario

Está claro que esta mujer precisará radioterapia, como siempre que realizamos una resección
conservadora. Por otra parte, también es necesario administrar poliquimioterapia, ya que existen
ganglios linfáticos con invasión tumoral. Sin embargo, no disponemos de datos sobre la presencia
o ausencia de receptores hormonales en el tumor, por lo que no podemos saber con estos datos si
es tributaria o no de recibir hormonoterapia.(R/1)

112. Si al examinar a una paciente con cáncer de cervix usted encuentra una “pelvis
congelada” y el ultrasonido le informan hidronefrosis izquierda, el estadio clínico
correcto sería:

1. 1. IIb.
2. 2. IIIb.
3. 3. IVa.
4. 4. IIIa.
Gráfico de respuestas
Comentario

 
 
 

Concepto poco preguntado en el ENARM, eventualmente preguntan estadiajes pero no es lo más


común te aconsejo que los repases una semana antes cuando mucho antes. Estadio IIIb de la
FIGO: tumor que invade la pared pélvica, que causa hidronefrosis o riñón no funcional. (R/2)

113. En la sala de partos se observa que un lactante tiene movimientos respiratorios, pero
que el aire no entra en los pulmones si la boca está cerrada. El diagnóstico más probable:

1. 1. Atresia de coanas.
2. 2. Hernia diafragmática.
3. 3. Hipoplasia pulmonar.
4. 4. Cardiopatía congénita.
Gráfico de respuestas
Comentario

Esta pregunta es bastante intuitiva. Están describiendo una apnea obstructiva (en las apneas
centrales, como la inducida por la narcosis, no hay excursiones torácicas). La única posibilidad de
apnea obstructiva descrita en la pregunta es la atresia de coanas. Vemos que el niño tiene
compromiso respiratorio cuando no puede utilizar la boca para respirar. En la hipoplasia pulmonar
por hernia diafragmática, el aire no entra en ningún caso, tenga la boca abierta o cerrada el
pequeño.(R/1)

114. La displasia broncopulmonar es el resultado de :

1. 1. Daño inflamatorio pulmonar en un feto en desarrollo.


2. 2. Falla en el desarrollo de las arteriolas pulmonares en el RN.
3. 3. Falla en el desarrollo bronquial del RN.
El uso de oxígeno y presión positiva durante el síndrome de dificultad
4. 4.
respiratoria-RN.
Gráfico de respuestas
Comentario

Pregunta sencilla sobre DBP. La respuesta correcta es la 4, ya que es causada por el uso de
oxígeno y presión positiva. Repase la siguiente tabla.

 
 
 

115. Los anticuerpos anti-Ro no se asocian a:

1. 1. Lupus cutaneo subagudo.


2. 2. Haplotipo DR-3.
3. 3. Lupus neonatal.
4. 4. Wolf Parkinson White en lactantes.
Gráfico de respuestas
Comentario

En el lupus eritematoso sistémico (LES) es característica la relación de diversas manifestaciones


clínicas con la presencia de algunos anticuerpos. Un ejemplo es lo que ocurre con los anticuerpos
anti-Ro que aparecen en un 30% de los pacientes con LES, también denominados SS-A. Se ha
visto que se relacionan con el lupus cutáneo subagudo, LES del anciano y lupus neonatal. En esta
última situación el paso de anticuerpos de la madre al feto durante la gestación puede producir en
el recien nacido lesiones cutáneas o bloqueo cardiaco congénito, pero no un WPW como indica la
respuesta número 4. La aparición de este anticuerpo es más habitual en los sujetos con HLA- DR3.
Además de en el LES es muy haitual la presencia de anticuerpos anti- Ro en el síndrome de
Sjögren (60%). (R/4)

116. Masculino de 45 años acude a su consulta por presentar hematuria. En la exploración


clínica se aprecia linfedema en miembros inferiores. El tacto rectal muestra próstata con
superficies irregulares laterales. Es cierto que:

1. 1. Tendrá un PSA elevado con baja concentración de fosfatasa ácida.


2. 2. Tendrá un PSA bajo con elevada fosfatasa ácida.
3. 3. Tendrá elevado el PSA y elevada fosfatasa ácida.
4. 4. Tendrá disminuida la enolasa neuroespecífica.
Gráfico de respuestas
Comentario

El caso que nos plantean corresponde, con bastante probabilidad, a un cáncer de próstata, ya que
el tacto rectal nos habla de una glándula de superficie irregular. Por ello, lo más probable es que el
PSA esté aumentado, como suele suceder en el cáncer de próstata. En cuanto a la fosfatasa ácida
prostática, recuerde que, cuando está elevada, suele deberse a afectación extraprostática. La
presencia de linfedema en miembros inferiores es sugerente de afectación de los ganglios
linfáticos regionales, por lo que cabría esperar el aumento de este marcador.(R/3)

117. El limite recomendado de 15 mmHg para el neumoperitoneo, ¿tiene relación con que
parámetro?

1. 1. Distensibilidad de la pared abdominal.


2. 2. Compresibilidad de la pared arterial.
3. 3. Presión venosa.
4. 4. Presión de perfusión tisular.
Gráfico de respuestas
Comentario
 
 

You might also like